Download OU Shoulder and Elbow OITE Review

Document related concepts

Dental emergency wikipedia , lookup

Transcript
OU Shoulder and Elbow OITE
Review
September 23, 2015
Betsy M. Nolan MD
CONFIDENTIAL
1
• A 45-year-old man who had gout in his foot 2 years ago has
a 3-day history of elbow pain without an injury. The pain is
diffuse, constant, and worse with any movement.
Examination shows motion from 20 degrees to 90 degrees.
There is no erythema around his elbow, he has no fever,
and a sensory and motor examination of his arm is
unremarkable. Radiographs only show an effusion. The
patient’s uric acid level is within defined limits. What is the
next diagnostic step?
• 1- Elbow joint aspiration
2- MRI scan
3- Splint for 2 weeks and repeat examination
4- Sedimentation rate and C-reactive protein level
CONFIDENTIAL
2
•
PREFERRED RESPONSE: 1- Elbow joint aspiration
•
•
DISCUSSION
The best way to make the diagnosis is to aspirate the fluid from the joint and send it to the
laboratory for a cell count and crystal search. This will allow for the diagnosis of an infection, gout,
or pseudogout. An MRI scan will confirm the examination finding of an effusion, but it will not
reveal the cause of an inflammatory effusion. If the patient has chronic gout, an MRI scan or
ultrasound can aid in diagnosis if gout tophi are seen. A splint will help relieve the pain from the
effusion and the effusion may resolve on its own, but it will not contribute to a diagnosis.
Sedimentation rate and C-reactive protein level will help to rule out an infection, but they are not
as diagnostic as an aspiration.
•
•
RECOMMENDED READINGS
Orzechowski NM, Mason TG. Seronegative inflammatory arthritis. In: Morrey BF, Sanchez-Sotelo J,
eds. The Elbow and Its Disorders. 4th ed. Philadelphia, PA: Saunders-Elsevier; 2009:1039-1041.
•
de Ávila Fernandes E, Kubota ES, Sandim GB, Mitraud SA, Ferrari AJ, Fernandes AR. Ultrasound
features of tophi in chronic tophaceous gout. Skeletal Radiol. 2011 Mar;40(3):309-15. Epub 2010 Jul
31. PubMed PMID: 20676636.
CONFIDENTIAL
3
A 65-year-old man who underwent an uncomplicated reverse total shoulder arthroplasty (rTSA) to treat rotator cuff arthropathy 2 years ago
has a routine follow-up visit in your clinic. A radiograph at 2-year followup is shown in Figure 2. He denies shoulder pain and dysfunction and
constitutional symptoms, and his clinical examination findings are benign. Based upon the present radiologic evaluation, what is the next
most appropriate step?
1- Revision rTSA
2- Conversion to hemiarthroplasty
3- Continued observation
4- Infection work-up with screening labs and joint aspiration
CONFIDENTIAL
4
•
PREFERRED RESPONSE: 3- Continued observation
•
•
DISCUSSION
Based upon the patient’s clinical examination and symptoms, continued observation is appropriate. The remaining
options are not indicated. The radiograph reveals scapular notching, one of the more common complications
specific to rTSA. Notching is caused by repeated contact between the humeral component and/or humerus and
the inferior pillar of the scapular neck. Generation of particulate debris from this interaction can result in
osteolysis with the potential for screw and base plate failure. The overall incidence of notching has been reported
to be between 51% and 96%. This nearly ubiquitous finding has been attributed to implant positioning, altered
glenoid and humeral anatomy, and duration of implantation. Recent studies that indicate increased lateral offset,
increased glenosphere size, and inferior positioning of the base plate may reduce the incidence of scapular
notching.
•
•
RECOMMENDED READINGS
Cheung E, Willis M, Walker M, Clark R, Frankle MA. Complications in reverse total shoulder arthroplasty. J Am Acad
Orthop Surg. 2011 Jul;19(7):439-49. Review. PubMed PMID: 21724923.
View Abstract at PubMed
Lévigne C, Boileau P, Favard L, Garaud P, Molé D, Sirveaux F, Walch G. Scapular notching in reverse shoulder
arthroplasty. J Shoulder Elbow Surg. 2008 Nov-Dec;17(6):925-35. Epub 2008 Jun 16. PubMed PMID: 18558499.
View Abstract at PubMed
Gutiérrez S, Levy JC, Frankle MA, Cuff D, Keller TS, Pupello DR, Lee WE 3rd. Evaluation of abduction range of
motion and avoidance of inferior scapular impingement in a reverse shoulder model. J Shoulder Elbow Surg. 2008
Jul-Aug;17(4):608-15. Epub 2008 Mar 6. PubMed PMID: 18325795.
•
•
CONFIDENTIAL
5
• A 61-year-old right-hand-dominant woman sustains a fall down 3
stairs, resulting in a left anteroinferior dislocation and
noncomminuted greater tuberosity fracture. A closed glenohumeral
reduction with intravenous sedation is performed in the emergency
department. After reduction, the greater tuberosity fragment
remains displaced by 2 mm. What is the most appropriate
treatment?
• 1- Open reduction internal fixation with transosseous sutures
2- Arthroscopic fixation using a suture bridge technique
3- Nonsurgical treatment with early passive range of motion
4- Nonsurgical treatment with sling immobilization for 4 weeks
CONFIDENTIAL
6
•
PREFERRED RESPONSE: 3- Nonsurgical treatment with early passive range of motion
•
•
DISCUSSION
Greater tuberosity fractures and rotator cuff tears associated with a traumatic dislocation are more
commonly seen in women older than age 60. Greater tuberosity fractures that are displaced less
than 5 mm in the general population and less than 3 mm in laborers and professional athletes can
be treated successfully without surgery. Early passive range of motion is important to avoid the
complication of stiffness.
•
•
RECOMMENDED READINGS
George MS. Fractures of the greater tuberosity of the humerus. J Am Acad Orthop Surg. 2007
Oct;15(10):607-13. Review. PubMed PMID: 17916784.
View Abstract at PubMed
Platzer P, Kutscha-Lissberg F, Lehr S, Vecsei V, Gaebler C. The influence of displacement on shoulder
function in patients with minimally displaced fractures of the greater tuberosity. Injury. 2005
Oct;36(10):1185-9. Epub 2005 Jun 16. PubMed PMID: 15963996.
View Abstract at PubMed
Mattyasovszky SG, Burkhart KJ, Ahlers C, Proschek D, Dietz SO, Becker I, Müller-Haberstock S,
Müller LP, Rommens PM. Isolated fractures of the greater tuberosity of the proximal humerus: a
long-term retrospective study of 30 patients . Acta Orthop. 2011 Dec;82(6):714-20. doi:
10.3109/17453674.2011.618912. Epub 2011 Sep 6. PubMed PMID: 21895502.
•
•
CONFIDENTIAL
7
CLINICAL SITUATION FOR QUESTIONS 5 AND 6
A 40-year-old riveter who works in a manufacturing plant experiences gradual onset of anterior shoulder pain that started 4 months ago,
and he now has a constant ache in his shoulder. His pain wakes him at night and is worsened by lifting anything at shoulder height. He
does not have any radiation of pain, and neck motion does not aggravate his pain. He stopped doing riveting work 1 month ago, but the
pain did not improve. Examination shows normal motion and strength, a positive impingement sign, and tenderness over the anterior
greater tuberosity. His sulcus sign is negative, and his Yergason and speed test findings are negative. He has normal scapular mechanics.
Question 5
What is the next most appropriate step to confirm the diagnosis?
1- Radiograph
2- MRI scan
3- Subacromial injection with lidocaine
4- Ultrasound
CONFIDENTIAL
8
• PREFERRED RESPONSE: 3- Subacromial
injection with lidocaine
CONFIDENTIAL
9
• Question 6
• He started physical therapy while continuing light duty at
work. Eight weeks later, his pain remained unchanged. An
MRI scan is shown in Figure 5. What histologic changes are
likely to be found in the supraspinatus tendon?
• 1- Disorganized collagen fibers with mucoid degeneration
2- Disorganized collagen fibers and acute inflammatory
cells
3- Normal tendon fibers infiltrated with capillary
proliferation
4- Normal tendon fibers infiltrated with acute inflammatory
cells
CONFIDENTIAL
10
•
•
•
•
PREFERRED RESPONSE: 1- Disorganized collagen fibers with mucoid degeneration
•
•
RECOMMENDED READINGS
Park HB, Yokota A, Gill HS, El Rassi G, McFarland EG. Diagnostic accuracy of clinical tests for the different degrees of subacromial impingement
syndrome. J Bone Joint Surg Am. 2005 Jul;87(7):1446-55. PubMed PMID: 15995110.
View Abstract at PubMed
Khan KM, Cook JL, Bonar F, Harcourt P, Astrom M. Histopathology of common tendinopathies. Update and implications for clinical management.
Sports Med. 1999 Jun;27(6):393-408. Review. PubMed PMID: 10418074.
View Abstract at PubMed
Lauder TD. Musculoskeletal disorders that frequently mimic radiculopathy. Phys Med Rehabil Clin N Am. 2002 Aug;13(3):469-85. Review. PubMed
PMID: 12380546.
View Abstract at PubMed
Cannon DE, Dillingham TR, Miao H, Andary MT, Pezzin LE. Musculoskeletal disorders in referrals for suspected cervical radiculopathy. Arch Phys Med
Rehabil. 2007 Oct;88(10):1256-9. PubMed PMID: 17908566.
•
•
•
DISCUSSION
This patient has impingement syndrome based on the history and examination. The best way to confirm the diagnosis is by performing a subacromial
injection with lidocaine, which is also called a Neer impingement test. If the pain is relieved, the patient’s pain is coming from the subacromial space.
An MRI scan would not confirm the diagnosis of impingement, although it can aid in diagnosis of other causes of anterior shoulder pain such as a
rotator cuff tear. This patient has normal rotator cuff strength, so that diagnosis is less likely. A radiograph can show acromial morphology, which
would support the diagnosis of impingement, but it does not rule out impingement if the radiograph findings are normal. Ultrasound would not
support the diagnosis of impingement, but, like an MRI scan, it can reveal pathologies other than impingement. The MRI scan shows a supraspinatus
tendon with changes consistent with tendinopathy, which is defined by disorganized collagen fibers with mucoid degeneration on the microscopic
level. Although there are always exceptions, most tendinopathy occurs without inflammatory cells or capillary proliferation.
CONFIDENTIAL
11
• CLINICAL SITUATION FOR QUESTIONS 7 THROUGH 9
• A 26-year-old man has the chief complaint of right shoulder instability. He
underwent an uncomplicated arthroscopic Bankart repair following an
injury sustained while playing high school football. His condition was
stable for 7 years, but he redislocated his shoulder in a fall 6 months ago.
He describes weekly anterior instability events that he can reduce on his
own. Radiographs reveal a located glenohumeral joint, but a Hill-Sachs
lesion is noted. A CT scan reveals a 20% anteroinferior glenoid deficiency
and a Hill-Sachs lesion involving 20% of the humeral head.
• Question 7
• What is the best surgical option for this patient?
• 1- Coracoid transfer
2- Open Bankart repair
3- Revision arthroscopic Bankart repair
4- Arthroscopic remplissage procedure
CONFIDENTIAL
12
• PREFERRED RESPONSE: 1- Coracoid transfer
CONFIDENTIAL
13
• Question 8
• What is the best indication to treat a Hill-Sachs
lesion?
• 1- A lesion involving 20% of the humeral head
that does not engage on examination
2- A lesion involving 25% of the humeral head
that remains located following instability repair
3- A lesion involving 30% of the humeral head
that engages on examination
4- A lesion involving 40% of the humeral head
with recurrent glenohumeral instability
CONFIDENTIAL
14
• PREFERRED RESPONSE: 4- A lesion involving
40% of the humeral head with recurrent
glenohumeral instability
CONFIDENTIAL
15
• Question 9
• What is the most likely predictor of
postsurgical pain following a coracoid transfer
procedure for recurrent shoulder instability?
• 1- Suboptimal graft placement
2- Pain before surgery
3- Progression of osteoarthritis
4- Previous surgical treatment
CONFIDENTIAL
16
•
PREFERRED RESPONSE: 2- Pain before surgery
•
•
DISCUSSION
Recurrent glenohumeral instability with anteroinferior glenoid bone loss is best treated with a coracoid transfer. Schmid and associates reported on
49 patients with these lesions and recurrent instability who underwent coracoid transfer procedures. Good and excellent outcomes were reported in
88% of patients, and there were no instances of recurrent instability. With anteroinferior glenoid bone loss, capsular procedures—whether open or
arthroscopic—are unlikely to provide stability. Remplissage is a procedure designed to address Hill-Sachs lesions rather than glenoid defects. The
absolute indications to treat Hill-Sachs lesions surgically include a lesion associated with a humeral head fracture, a lesion involving more than 30% of
the humeral head with chronic instability, and reverse Hill-Sachs lesions involving more than 20% of the articular surface with symptoms of posterior
instability. Lesions involving 20% to 35% with or without engagement on examination are relative indications, as are lesions exceeding 10% that do
not remain centered in the glenoid following arthroscopic stabilization.
In Schmid and associates’ series of coracoid transfers for recurrent instability and anterior glenoid deficiency, patients who reported pain before
surgery were 20 times more likely to have pain after surgery that compromised the functional outcome. Optimal graft placement correlated with
better functional outcomes and less progression of arthrosis, but not with pain. Consequently, poor graft position, arthritis progression, and prior
surgical treatment are not as consistently predictive of pain after surgery.
•
•
•
•
•
RECOMMENDED READINGS
Schmid SL, Farshad M, Catanzaro S, Gerber C. The Latarjet procedure for the treatment of recurrence of anterior instability of the shoulder after
operative repair: a retrospective case series of forty-nine consecutive patients. J Bone Joint Surg Am. 2012 Jun 6;94(11):e75. doi:
10.2106/JBJS.K.00380. PubMed PMID: 22637215.
View Abstract at PubMed
Provencher MT, Frank RM, Leclere LE, Metzger PD, Ryu JJ, Bernhardson A, Romeo AA. The Hill-Sachs lesion: diagnosis, classification, and
management. J Am Acad Orthop Surg. 2012 Apr;20(4):242-52. doi: 10.5435/JAAOS-20-04-242. Review. PubMed PMID: 22474094.
View Abstract at PubMed
Boileau P, O'Shea K, Vargas P, Pinedo M, Old J, Zumstein M. Anatomical and functional results after arthroscopic Hill-Sachs remplissage. J Bone Joint
Surg Am. 2012 Apr 4;94(7):618-26. doi: 10.2106/JBJS.K.00101. PubMed PMID: 22488618.
CONFIDENTIAL
17
• Question 10
• A 45-year-old woman with diabetes has a 3-month history of left
shoulder pain and motion loss unrelated to trauma. She previously
underwent treatment with nonsteroidal anti-inflammatory
medication and a home stretching program, experiencing minimal
relief of her symptoms. Examination reveals loss of passive external
rotation, abduction, and forward elevation without reduction in
strength. Radiograph findings are normal. What is the most
appropriate next step?
• 1- MRI scan
2- Cortisone injection therapy with continued physical therapy (PT)
3- Closed manipulation under anesthesia
4- Arthroscopic release with manipulation under anesthesia
CONFIDENTIAL
18
•
PREFERRED RESPONSE: 2- Cortisone injection therapy with continued physical therapy (PT)
•
•
DISCUSSION
Based upon the duration of symptoms and clinical presentation, this patient would benefit from cortisone injection therapy and continued PT.
Adhesive capsulitis is most commonly an idiopathic process that results in joint pain and loss of motion from capsular contracture. It affects
approximately 2% to 5% of the general population. The process typically affects middle-age women. There are secondary causes such as previous
trauma and fractures as well as associated medical conditions such as diabetes, stroke, and cardiac and thyroid disease. Debate remains as to
whether there is a genetic predisposition for the development of adhesive capsulitis despite increased frequency noted in twin studies. Although the
underlying etiology and pathophysiology are not well understood, the consensus is that synovial inflammation and capsular fibrosis result in pain and
joint volume loss. It is hypothesized that in patients with diabetes, an increased rate of glycosylation and cross-linking of the shoulder capsule raises
the incidence of frozen shoulder. For this patient, history reveals a short course of symptoms that did not improve with nonsurgical modalities.
Clinically, the patient has reduced passive range of motion, particularly with external and internal rotation and forward elevation. Radiographs are
usually obtained to exclude other causes of shoulder pain such as glenohumeral arthrosis, malignancy, calcific tendonitis, impingement, and
acromioclavicular degeneration. If pain and stiffness persist beyond 6 months, closed manipulation may be an option. Complications associated with
this modality may include humerus fracture, dislocation, hematoma, rotator cuff and labral tears, and brachial plexus injury. Some surgeons advocate
arthroscopic capsular release to allow for examination of concomitant pathology and controlled release of capsular tissue, with the potential for
reduced required force when performing the manipulation portion of the procedure. This modality may be appropriate after an initial treatment with
PT. Controversy remains as to whether posterior capsular release should be performed routinely because studies have shown outcomes to be similar
with anterior and combined approaches. Therapy should be initiated early after intervention, with some surgeons advocating admission to the
hospital with inpatient therapy for pain management and compliance.
•
•
RECOMMENDED READINGS
Neviaser AS, Neviaser RJ. Adhesive capsulitis of the shoulder. J Am Acad Orthop Surg. 2011 Sep;19(9):536-42. Review. PubMed PMID: 21885699.
View Abstract at PubMed
Hsu JE, Anakwenze OA, Warrender WJ, Abboud JA. Current review of adhesive capsulitis. J Shoulder Elbow Surg. 2011 Apr;20(3):502-14. Epub 2010
Dec 16. Review. PubMed PMID: 21167743.
View Abstract at PubMed
Le Lievre HM, Murrell GA. Long-term outcomes after arthroscopic capsular release for idiopathic adhesive capsulitis. J Bone Joint Surg Am. 2012 Jul
3;94(13):1208-16. PubMed PMID: 22760389.
View Abstract at PubMed
Yian EH, Contreras R, Sodl JF. Effects of glycemic control on prevalence of diabetic frozen shoulder. J Bone Joint Surg Am. 2012 May 16;94(10):91923. PubMed PMID: 22617920.
•
•
•
CONFIDENTIAL
19
• A 42-year-old woman has a posterior elbow
dislocation. Closed reduction is performed, and the
elbow appears stable under fluoroscopic examination.
Further treatment should consist of
• 1- early mobilization only.
2- surgical repair or reconstruction of the lateral
collateral ligament (LCL) and the medial collateral
ligament (MCL).
3- active range of motion in a hinged brace with a
range of 30 degrees to 120 degrees.
4- application of a hinged external fixator with early
mobilization.
CONFIDENTIAL
20
•
PREFERRED RESPONSE: 1- early mobilization only.
•
•
DISCUSSION
This is a simple (no associated fracture) elbow dislocation. Such dislocations can be treated with closed reduction
followed by mobilization after 5 to 7 days to avoid stiffness, provided the elbow is stable through a full arc of
motion at the time of reduction. If the elbow is unstable but has a short arc of stability, then using a hinged brace
in the stable arc may be considered. (Note: It may be necessary to splint the elbow in pronation if the MCL is
intact and the LCL is disrupted or in supination if the LCL is intact but the MCL disrupted.) Surgical reconstruction
of the LCL and MCL may be required only if the elbow does not have a stable arc at the time of reduction. If
unstable after reconstruction, application of a hinged external fixator may be considered.
•
•
RECOMMENDED READINGS
Glaser D, Armstrong A. Elbow and forearm trauma. In: Flynn J ed. Orthopaedic Knowledge Update 10. Rosemont,
IL: American Academy of Orthopaedic Surgeons; 2011:325-341.
•
Anakwe RE, Middleton SD, Jenkins PJ, McQueen MM, Court-Brown CM. Patient-reported outcomes after simple
dislocation of the elbow. J Bone Joint Surg Am. 2011 Jul 6;93(13):1220-6. doi: 10.2106/JBJS.J.00860. PubMed
PMID: 21776575.
View Abstract at PubMed
Mehlhoff TL, Noble PC, Bennett JB, Tullos HS. Simple dislocation of the elbow in the adult. Results after closed
treatment. J Bone Joint Surg Am. 1988 Feb;70(2):244-9. PubMed PMID: 3343270.
•
CONFIDENTIAL
21
• A 65-year-old woman has 4 months of atraumatic shoulder
pain persisting despite physical therapy and activity
modification. She has normal range of motion, and an MRI
scan reveals a 10% thickness partial articular supraspinatus
tear.
• 1- Physical therapy and activity modification
2- Corticosteroid injection
3- Arthroscopic glenohumeral capsular release
4- Arthroscopic superior labrum anterior to posterior
(SLAP) repair
5- Arthroscopic subacromial decompression and rotator
cuff debridement
6- Arthroscopic subacromial decompression and rotator
cuff repair
CONFIDENTIAL
22
• PREFERRED RESPONSE: 2- Corticosteroid
injection
CONFIDENTIAL
23
• A 35-year-old mechanic has 6 months of shoulder pain
following an axial traction work-related injury. His pain has
persisted despite extensive physical therapy and work
restrictions. A noncontrast MRI scan shows a 90% partial
bursal-sided supraspinatus tear.
• 1- Physical therapy and activity modification
2- Corticosteroid injection
3- Arthroscopic glenohumeral capsular release
4- Arthroscopic superior labrum anterior to posterior
(SLAP) repair
5- Arthroscopic subacromial decompression and rotator
cuff debridement
6- Arthroscopic subacromial decompression and rotator
cuff repair
CONFIDENTIAL
24
• PREFERRED RESPONSE: 6- Arthroscopic
subacromial decompression and rotator cuff
repair
CONFIDENTIAL
25
• A 25-year-old 100% service connected Veteran
has a type II SLAP lesion, with 4 weeks of newonset atraumatic shoulder pain.
• 1- Physical therapy and activity modification
2- Corticosteroid injection
3- Arthroscopic superior labrum anterior to
posterior (SLAP) repair
4- Arthroscopic subacromial decompression and
rotator cuff debridement
5- Email your shipmates to ask for their help.
CONFIDENTIAL
26
CONFIDENTIAL
27
• PREFERRED RESPONSE: 1- Physical therapy
and activity modification
CONFIDENTIAL
28
• A 75-year-old man has had 8 months of persistent, atraumatic
shoulder pain. He had transient improvement with physical therapy
and a subacromial corticosteroid injection. MRI scan shows a 25%
partial articular supraspinatus/subscapularis tear and significant
subacromial bursal inflammation.
• 1- Physical therapy and activity modification
2- Corticosteroid injection
3- Arthroscopic glenohumeral capsular release
4- Arthroscopic superior labrum anterior to posterior (SLAP) repair
5- Arthroscopic subacromial decompression and rotator cuff
debridement
6- Arthroscopic subacromial decompression and rotator cuff repair
CONFIDENTIAL
29
•
PREFERRED RESPONSE: 5- Arthroscopic subacromial decompression and rotator cuff debridement
•
•
DISCUSSION
The management of partial rotator cuff tears depends upon many factors, including patient age,
symptom onset and severity, prior treatment, physical limitation(s) based on history and
examination, and the extent of structural involvement based upon detailed imaging (typically MRI
or ultrasound). The incidence of a partial rotator cuff tear, based on imaging with MRI or
ultrasound, is high in patients ages 60 years and older. Patients beyond age 60 with either mild or
new-onset symptoms with preserved active and passive range of motion are excellent candidates
for physical therapeutic intervention and avoidance of exacerbating activities, particularly when
MRI scan or ultrasound reveal less than 50% tendon involvement. Partial rotator cuff tears are also
common in the dominant arm of overhead athletes, and frequently respond to nonsurgical
treatment, as well. These types of partial rotator cuff tears often are seen in combination with
superior labral pathology. Rotator cuff repair usually is recommended for patients with tears that
involve more than 50% of tendon thickness who have failed a reasonable attempt at nonsurgical
management, particularly patients who are young and have high activity demands. Partial-sided
bursal tears may be more symptomatic and respond well to surgical repair, but patients may not do
as well with subacromial decompression alone. Global loss of glenohumeral motions is consistent
with adhesive capsulitis. Such patients are initially treated with therapy that emphasizes range of
motion, usually incorporating a home exercise program. Finally, subacromial decompression may be
considered for patients with low-grade partial articular rotator cuff tears that have failed
nonsurgical management and substantially interfere with daily and/or recreational activities.
CONFIDENTIAL
30
•
•
•
•
•
•
•
•
RECOMMENDED READINGS
Sher JS, Uribe JW, Posada A, Murphy BJ, Zlatkin MB. Abnormal findings on magnetic resonance images of asymptomatic shoulders. J Bone Joint Surg
Am. 1995 Jan;77(1):10-5. PubMed PMID: 7822341.
View Abstract at PubMed
Weber SC. Arthroscopic debridement and acromioplasty versus mini-open repair in the treatment of significant partial-thickness rotator cuff tears.
Arthroscopy. 1999 Mar;15(2):126-31. PubMed PMID: 10210067.
View Abstract at PubMed
Cordasco FA, Backer M, Craig EV, Klein D, Warren RF. The partial-thickness rotator cuff tear: is acromioplasty without repair sufficient? Am J Sports
Med. 2002 Mar-Apr;30(2):257-60. PubMed PMID: 11912097.
View Abstract at PubMed
Connor PM, Banks DM, Tyson AB, Coumas JS, D'Alessandro DF. Magnetic resonance imaging of the asymptomatic shoulder of overhead athletes: a 5year follow-up study. Am J Sports Med. 2003 Sep-Oct;31(5):724-7. PubMed PMID: 12975193.
View Abstract at PubMed
Yamaguchi K, Ditsios K, Middleton WD, Hildebolt CF, Galatz LM, Teefey SA. The demographic and morphological features of rotator cuff disease. A
comparison of asymptomatic and symptomatic shoulders. J Bone Joint Surg Am. 2006 Aug;88(8):1699-704. PubMed PMID: 16882890.
View Abstract at PubMed
Loeffler BJ, Brown SL, D'Alessandro DF, Fleischli JE, Connor PM. Incidence of False Positive Rotator Cuff Pathology in MRIs of Patients with Adhesive
Capsulitis. Orthopedics. 2011 May 18;34(5):362. doi: 10.3928/01477447-20110317-14. PubMed PMID: 21598899.
View Abstract at PubMed
Pedowitz RA, Yamaguchi K, Ahmad CS, Burks RT, Flatow EL, Green A, Iannotti JP, Miller BS, Tashjian RZ, Watters WC 3rd, Weber K, Turkelson CM,
Wies JL, Anderson S, St Andre J, Boyer K, Raymond L, Sluka P, McGowan R; American Academy of Orthopaedic Surgeons. Optimizing the management
of rotator cuff problems. J Am Acad Orthop Surg. 2011 Jun;19(6):368-79. PubMed PMID: 21628648.
CONFIDENTIAL
31
The problem seen above is most likely associated with injury to what structure?
1- Inferior glenohumeral ligament
2- Acromioclavicular (AC) ligaments
3- Coracoclavicular ligaments
4- Coracoacromial ligament
5. Pocket protector
CONFIDENTIAL
32
•
PREFERRED RESPONSE: 3- Coracoclavicular ligaments
•
•
DISCUSSION
The radiograph shows an extra-articular distal clavicle fracture lateral to the clavicular attachment point of the
coracoclavicular ligaments (conoid and trapezoid). However, unlike a scenario featuring a typical Neer type I
fracture, the interval between coracoid and clavicle is clearly widened and there is marked fracture displacement.
It is clear that the coracoclavicular ligaments must also be torn. The inferior glenohumeral ligament is important to
glenohumeral joint stability, but has no effect on the relationship between clavicle and scapula. The AC ligaments
are thickenings of the AC joint capsule. They have been shown to be responsible for 90% of anteroposterior
stability of the AC joint. The coracoclavicular ligaments are responsible for 77% of stability for superior translation
(as in this case). The coracoacromial ligament connects 2 parts of the scapula (coracoids and acromion) and is part
of the arch that supports the rotator cuff.
•
•
RECOMMENDED READINGS
NEER CS 2nd. Fracture of the distal clavicle with detachment of the coracoclavicular ligaments in adults. J Trauma.
1963 Mar;3:99-110. PubMed PMID: 13937900.
View Abstract at PubMed
URIST MR. COMPLETE DISLOCATION OF THE ACROMIOCLAVICULAR JOINT. J Bone Joint Surg Am. 1963
Dec;45:1750-3. PubMed PMID: 14083156.
View Abstract at PubMed
Acus RW 3rd, Bell RH, Fisher DL. Proximal clavicle excision: an analysis of results. J Shoulder Elbow Surg. 1995 MayJun;4(3):182-7. PubMed PMID: 7552675.
•
•
CONFIDENTIAL
33
CONFIDENTIAL
34
• A 36-year-old right-hand-dominant man fell from his
motorcycle and sustained the acute right upper extremity
injury seen. At surgery, an open reduction and internal
fixation of the ulna is performed along with attempted
open reduction of the radiocapitellar joint. However, the
radial head is slightly subluxed in flexion and redislocates
with elbow extension below 90 degrees. What is the most
appropriate treatment at this time?
• 1- Radial head resection
2- Casting in 90 degrees of flexion for 3 weeks, followed by
reassessment of elbow stability
3- Repair of the annular ligament
4- Revision open reduction and internal fixation of the ulnar
fracture
CONFIDENTIAL
35
•
PREFERRED RESPONSE: 4- Revision open reduction and internal fixation of the ulnar fracture
•
•
DISCUSSION
This case is a variant of a type I Monteggia fracture according to the Bado classification with a
segmental ulna fracture. In some cases, the radial head subluxation can be subtle, and missing this
would lead to a poor outcome. In this case, the anterior radial head dislocation is obvious, but the
segmental nature of the ulna fracture makes anatomic reduction difficult. The radial head usually
spontaneously reduces once the ulna is anatomically reduced, and no surgical treatment to the
lateral side is required. When this is not the case, a lateral approach and incision of the annular
ligament may be required for reduction. If an open reduction of the radial head is unsuccessful, the
problem is almost always residual malalignment of the ulna. Therefore, casting and annular
ligament repair will not improve reduction. A radial head resection would eliminate the
nonconcentric contact between radial head and capitellum, but would not be an appropriate
treatment for this young patient who has an acute, correctable fracture deformity.
•
•
RECOMMENDED READINGS
Bado JL. The Monteggia lesion. Clin Orthop Relat Res. 1967 Jan-Feb;50:71-86. PubMed PMID:
6029027.
View Abstract at PubMed
Ring D, Jupiter JB, Waters PM. Monteggia fractures in children and adults. J Am Acad Orthop Surg.
1998 Jul-Aug;6(4):215-24. Review. PubMed PMID: 9682084.
•
CONFIDENTIAL
36
CONFIDENTIAL
37
• Figure 19 is the radiograph of a 45-year-old right-handdominant man who has had a 2-day history of right
shoulder pain, weakness, and a deformity involving the
clavicle region after a fall from a scaffold during work
activities. He was previously evaluated by his primary care
physician and another orthopaedist. He has sought a
second opinion regarding his treatment options. What is
the most appropriate treatment for his injury?
• 1- Sling immobilization with continued observation
2- Open reduction and plate fixation
3- Open reduction, plate fixation, and application of an
external bone stimulator
4- External fixation
CONFIDENTIAL
38
•
PREFERRED RESPONSE: 2- Open reduction and plate fixation
•
•
DISCUSSION
Midshaft clavicle fractures are relatively common and account for 3% to 10% of all adult fractures. Historical studies regarding nonsurgical treatment
of displaced clavicle fractures indicated a low nonunion rate approaching 1%. Contemporary studies, however, suggest that the nonunion rate is
much higher, reaching 15% to 20% with corresponding loss of shoulder strength and residual deficits at 6 months from date of injury. A recent metaanalysis by McKee and associates comparing nonsurgical treatment with a sling and surgical treatment with plate fixation concluded that initial
fixation of displaced midshaft clavicle fractures demonstrated a positive effect on pain reduction at 3 weeks and improved Disabilities of the Arm,
Shoulder, and Hand (DASH) scores at 6 weeks and 3 months after surgery. The choice of fixation remains a matter of debate because both plate and
pin fixation have been used to achieve clavicle union. Intramedullary fixation of clavicle fractures historically has demonstrated an increased risk for
pin tract infections and hardware removal attributable to local irritation from the implant. External bone stimulator use has not proven beneficial in
effecting a reduction of nonunion rates. The most common complications noted with surgical intervention included local hardware irritation and
wound infection.
•
•
RECOMMENDED READINGS
McKee RC, Whelan DB, Schemitsch EH, McKee MD. Operative versus nonoperative care of displaced midshaft clavicular fractures: a meta-analysis of
randomized clinical trials. J Bone Joint Surg Am. 2012 Apr 18;94(8):675-84. Review. PubMed PMID: 22419410.
View Abstract at PubMed
Canadian Orthopaedic Trauma Society. Nonoperative treatment compared with plate fixation of displaced midshaft clavicular fractures. A
multicenter, randomized clinical trial. J Bone Joint Surg Am. 2007 Jan;89(1):1-10. PubMed PMID: 17200303.
View Abstract at PubMed
Judd DB, Pallis MP, Smith E, Bottoni CR. Acute operative stabilization versus nonoperative management of clavicle fractures. Am J Orthop (Belle
Mead NJ). 2009 Jul;38(7):341-5. PubMed PMID: 19714275.
•
•
CONFIDENTIAL
39
• A 55-year-old woman develops posttraumatic
arthritis in the elbow following a distal humerus
fracture. What is the most likely mid-term (5-10
years after surgery) complication following
semiconstrained total elbow arthroplasty (TEA)?
• 1- Bushing wear
2- Infection
3- Aseptic component loosening
4- Component fracture
CONFIDENTIAL
40
•
PREFERRED RESPONSE: 1- Bushing wear
•
•
DISCUSSION
TEA has been described for posttraumatic arthritis of the elbow and typically involves a young patient population
with multiple previous operations on the affected elbow. Morrey and Schneeberger found aseptic component
loosening to be uncommon (fewer than 10% of patients), and usually occurs more than 10 years after surgery.
Prosthetic fracture, usually of the ulnar component, is also a late-term finding. Infection is the most common
mode of early failure, but usually occurs within the first 5 years and has an overall rate of approximately 5%.
Bushing wear has been reported as the most common cause of mechanical TEA failure in this population at
intermediate-term followup.
•
•
RECOMMENDED READINGS
Schneeberger AG, Adams R, Morrey BF. Semiconstrained total elbow replacement for the treatment of posttraumatic osteoarthrosis. J Bone Joint Surg Am. 1997 Aug;79(8):1211-22. PubMed PMID: 9278082.
View Abstract at PubMed
Morrey BF, Schneeberger AG. Total elbow arthroplasty for posttraumatic arthrosis. Instr Course Lect. 2009;58:495504. PubMed PMID: 19385558.
View Abstract at PubMed
Throckmorton T, Zarkadas P, Sanchez-Sotelo J, Morrey B. Failure patterns after linked semiconstrained total elbow
arthroplasty for posttraumatic arthritis. J Bone Joint Surg Am. 2010 Jun;92(6):1432-41. doi: 10.2106/JBJS.I.00145.
PubMed PMID: 20516319.
•
•
CONFIDENTIAL
41
•
•
CLINICAL SITUATION FOR QUESTIONS 21 AND 22
A 21-year-old rugby player has had the sensation of shoulder instability while
making tackles for 3 years. Two years ago, he had an arthroscopic Bankart repair
and capsulorrhaphy that used 3 suture anchors after dislocating his shoulder while
making a tackle. This procedure required an emergency department sedated
reduction. After this dislocation, he had paresthesias in his arm and a sense of
weakness. His numbness eventually resolved. He did well after surgery until 2
weeks ago, when he again felt his shoulder dislocate while tackling and had an
emergency department reduction.
•
•
•
Question 21
What caused his recurrent instability?
1- The use of suture anchors in his repair
2- The physical therapy program after surgery
3- His age at the time of first surgery
4- His activity levels after surgery
5-Wearing packs when he was in the Army
•
CONFIDENTIAL
42
• PREFERRED RESPONSE: 4- His activity levels
after surgery
CONFIDENTIAL
43
• Question 22
• Numbness after his first dislocation was
related to
• 1- intrasurgical traction on the
musculocutaneous nerve.
2- residual interscalene blockade.
3- ulnar neuropathy after sling use.
4- sensory axillary nerve palsy from his
dislocation.
CONFIDENTIAL
44
•
PREFERRED RESPONSE: 4- sensory axillary nerve palsy from his dislocation.
•
•
DISCUSSION
Several studies have demonstrated the equivalency of arthroscopic Bankart repair to open surgery,
but most studies have also demonstrated increased recurrence rates in patients who participate in
collision sports such as rugby. Revision surgery with arthroscopy is unlikely to be durable, and in
patients who have large glenoid defects, a coracoid process transfer (Latarjet or Bristow) is the
preferred surgical treatment. Arthroplasty would not be indicated in a young and active patient.
•
•
RECOMMENDED READINGS
Neyton L, Young A, Dawidziak B, Visona E, Hager JP, Fournier Y, Walch G. Surgical treatment of
anterior instability in rugby union players: clinical and radiographic results of the Latarjet-Patte
procedure with minimum 5-year follow-up. J Shoulder Elbow Surg. 2012 Dec;21(12):1721-7. doi:
10.1016/j.jse.2012.01.023. Epub 2012 May 5. PMID 22565042.
View Abstract at PubMed
Robinson CM, Shur N, Sharpe T, Ray A, Murray IR. Injuries associated with traumatic anterior
glenohumeral dislocations. J Bone Joint Surg Am. 2012 Jan 4;94(1):18-26. PMID: 22218378.
•
CONFIDENTIAL
45
Figure 23a
Figure 23b
Figure 23c
Figure 23d
Question 23 of 105
Figures 23a through 23d are the radiographs and MRI scans of a 30-year-old otherwise healthy
man who sustained an anterior right shoulder dislocation while playing baseball. He requires a
closed reduction under sedation at a local emergency department. He is placed into a shoulder
immobilizer and referred to your office for further treatment. Upon inquiry, the patient states that
he previously dislocated the shoulder twice within the last year while playing basketball. He
demonstrates positive apprehension and speed tests. What is the most appropriate next treatment
step?
1- Brief period of immobilization with initiation of therapy
2- Arthroscopic labral repair
3- Open capsular shift
4- Coracoid transfer
CONFIDENTIAL
46
•
PREFERRED RESPONSE: 2- Arthroscopic labral repair
•
•
DISCUSSION
By history, this patient has recurrent anterior instability. His radiographs do not indicate significant deficiency of
the anterior glenoid, which would potentially require augmentation with a coracoid transfer to restore stability to
the glenohumeral joint. An MRI scan reveals a displaced anteroinferior labral tear (Bankart lesion) with extension
into the biceps labral complex. An open capsular shift can address capsular redundancy, but an arthroscopic
procedure will allow for examination of concomitant pathology and the ability to address the capsulolabral tear
with reduced morbidity. To minimize this patient's redislocation risk with sports activities, an arthroscopic
capsulolabral repair involving the anteroinferior and superior labrum is the most appropriate solution. Hantes and
associates demonstrated that clinical outcomes are similar at 2-year followup in patients with combined
anteroinferior and superior labral tears vs patients with isolated Bankart lesions when treated by arthroscopic
means.
•
•
RECOMMENDED READINGS
Carreira DS, Mazzocca AD, Oryhon J, Brown FM, Hayden JK, Romeo AA. A prospective outcome evaluation of
arthroscopic Bankart repairs: minimum 2-year follow-up. Am J Sports Med. 2006 May;34(5):771-7. PubMed PMID:
16627629.
Taylor DC, Arciero RA. Pathologic changes associated with shoulder dislocations. Arthroscopic and physical
examination findings in first-time, traumatic anterior dislocations. Am J Sports Med. 1997 May-Jun;25(3):306-11.
PubMed PMID: 9167808.
Hantes ME, Venouziou AI, Liantsis AK, Dailiana ZH, Malizos KN. Arthroscopic repair for chronic anterior shoulder
instability: a comparative study between patients with Bankart lesions and patients with combined Bankart and
superior labral anterior posterior lesions. Am J Sports Med. 2009 Jun;37(6):1093-8. Epub 2009 Mar 13. PubMed
PMID: 19286910.
•
CONFIDENTIAL
47
Figure 27
Figure 27 is the MRI scan of a 63-year-old man who has experienced 3 years of shoulder pain. He has had 2 fluoroscopically
guided corticosteroid injections that provided him with temporary but significant relief. His primary care physician ordered an
MRI scan because of his ongoing shoulder issues. His examination shows significant loss of motion in all planes but good motor
strength. The best treatment at this point would include
1- hyaluronic acid injection.
2- physical therapy.
3- reverse total shoulder arthroplasty.
4- anatomic total shoulder arthroplasty.
CONFIDENTIAL
48
•
PREFERRED RESPONSE: 4- anatomic total shoulder arthroplasty.
•
•
DISCUSSION
Glenohumeral arthritis in this age group is best treated with total shoulder arthroplasty. It provides more durable
and cost-effective relief than hemiarthroplasty. Hyaluronic acid has been demonstrated in some studies to be
effective at improving pain in the short term, but is not approved by the U.S. Food and Drug Administration for use
in the shoulder. Reverse total shoulder replacement is indicated in patients who have an irreparable rotator cuff
tear. This image clearly shows the supraspinatus is intact. Physical therapy has not proven beneficial in the setting
of end-stage glenohumeral arthritis.
•
•
RECOMMENDED READINGS
Izquierdo R, Voloshin I, Edwards S, Freehill MQ, Stanwood W, Wiater JM, Watters WC 3rd, Goldberg MJ, Keith M,
Turkelson CM, Wies JL, Anderson S, Boyer K, Raymond L, Sluka P; Treatment of glenohumeral osteoarthritis.
American Academy of Orthopedic Surgeons. J Am Acad Orthop Surg. 2010 Jun;18(6):375-82. PMID: 20511443.
Mather RC 3rd, Watters TS, Orlando LA, Bolognesi MP, Moorman CT 3rd. Cost effectiveness analysis of
hemiarthroplasty and total shoulder arthroplasty. J Shoulder Elbow Surg. 2010 Apr;19(3):325-34. PMID: 20303459.
Edwards TB, Kadakia NR, Boulahia A, Kempf JF, Boileau P, Némoz C, Walch G. A comparison of hemiarthroplasty
and total shoulder arthroplasty in the treatment of primary glenohumeral osteoarthritis: results of a multicenter
study. J Shoulder Elbow Surg. 2003 May-Jun;12(3):207-13. PMID: 12851570.
Gartsman GM, Roddey TS, Hammerman SM. Shoulder arthroplasty with or without resurfacing of the glenoid in
patients who have osteoarthritis. J Bone Joint Surg Am. 2000 Jan;82(1):26-34. PMID: 10653081.
•
•
CONFIDENTIAL
49
Figure 28
CLINICAL SITUATION FOR QUESTIONS 28 AND 29
Figure 28 is the radiograph of a 59-year-old woman who fell from a ladder 6 weeks ago when a tree her brother was cutting down
accidentally hit the ladder. After being evaluated for additional trauma, the emergency room physician ordered a VA “standard”
shoulder series, which consists of an AP in neutral rotation, AP in external rotation and AP in internal rotation. The radiologist read
this series as normal, and the patient was discharged home. She is now referred to your office for left shoulder pain and loss of
external rotation.
Question 28
What is the most likely diagnosis?
1- Rotator cuff tear
2- Adhesive capsulitis
3- Brachial plexus injury
4- Posterior shoulder dislocation
CONFIDENTIAL
50
• PREFERRED RESPONSE: 4- Posterior shoulder
dislocation
CONFIDENTIAL
51
• Question 29
• What is the best next step?
• 1- Physical therapy
2- CT scan
3- Arthroscopic capsular release
4- Arthroscopic Bankart repair
CONFIDENTIAL
52
•
PREFERRED RESPONSE: 2- CT scan
•
•
DISCUSSION
The patient has a posterior glenohumeral dislocation, as evidenced by the overlap on the initial radiograph. While
posterior dislocations are rare, they can be overlooked. A CT scan will accurately show the lesion prior to a trip to
the operating room. If a simple closed reduction is performed acutely and the arm is stable after the reduction, no
further intervention may be needed and treatment can be successful with a 2-week period of immobilization for
defects involving less than 30% of the humeral head. However, in this scenario, open reduction is likely and
stabilization may require a modified McLaughlin procedure or other intervention to fill in the humeral defect.
Younger male patients, those with a large humeral head defect, and those with seizure disorder may be at highest
risk for recurrence. For treatment of chronic posterior dislocations, it may be necessary to perform shoulder
arthroplasty to restore stability. Stiffness is attributable to articular incongruity; therefore, physical therapy and
capsular release are inappropriate.
•
•
RECOMMENDED READINGS
Duralde XA, Fogle EF. The success of closed reduction in acute locked posterior fracture-dislocations of the
shoulder. J Shoulder Elbow Surg. 2006 Nov-Dec;15(6):701-6. Epub 2006 Oct 19. PubMed PMID: 17055305.
Robinson CM, Aderinto J. Posterior shoulder dislocations and fracture-dislocations. J Bone Joint Surg Am. 2005
Mar;87(3):639-50. Review. PubMed PMID: 15741636.
Robinson CM, Seah M, Akhtar MA. The epidemiology, risk of recurrence, and functional outcome after an acute
traumatic posterior dislocation of the shoulder. J Bone Joint Surg Am. 2011 Sep 7;93(17):1605-13. doi:
10.2106/JBJS.J.00973. PubMed PMID: 21915575.
Sperling JW, Pring M, Antuna SA, Cofield RH. Shoulder arthroplasty for locked posterior dislocation of the shoulder.
J Shoulder Elbow Surg. 2004 Sep-Oct;13(5):522-7. PubMed PMID: 15383808.
•
CONFIDENTIAL
53
• A 40-year-old right-hand-dominant construction worker has a 3-month
history of right shoulder weakness secondary to a fall from a ladder at
work. He underwent nonsurgical treatment with anti-inflammatory
medication, cortisone injections, and therapy, with minimal relief of his
symptoms. A subsequent MRI scan indicates a 1-cm full-thickness
supraspinatus tendon tear. He has been referred to your clinic for
discussion of surgical intervention. The patient's nurse case manager is
concerned that he may not be able to return to his preinjury level of
activity at work, even with surgical intervention. You tell the nurse case
manager that, on average, the patient will
• 1- be at increased risk for infection compared to patients without a
Worker’s Compensation claim.
2- have significant functional improvement after rotator cuff repair that is
less robust than that of patients without a Worker’s Compensation claim.
3- have pain relief that is equivalent to that of patients without a Worker’s
Compensation claim.
4- return to work without restrictions within a 3-month time frame.
CONFIDENTIAL
54
•
PREFERRED RESPONSE: 2- have significant functional improvement after rotator cuff repair that is less robust than
that of patients without a Worker’s Compensation claim.
•
•
DISCUSSION
Many factors have been associated with less-than-favorable outcomes following rotator cuff repair such as tear
size, age at time of intervention, gender, fatty degeneration of rotator cuff musculature, and Worker’s
Compensation status. Henn and associates performed a prospective study regarding rotator cuff repairs in a
cohort of 125 patients to assess the factors that may affect outcome as measured with the Simple Shoulder Test
(SST), Disabilities of the Arm, Shoulder, and Hand (DASH), Short Form-36 (SF-36), and Visual Analog Pain Scale
(VAS). When confounding factors were controlled, Worker’s Compensation status was an independent predictor of
poorer DASH scores. With the use of historical controls, Bhatia and associates concluded that the vast majority
(89%) of workers who underwent an arthroscopic rotator cuff repair returned to their preoperative level of work
at a mean time of 7.6 months. There was a trend toward decreased return to full duty with increased work
demands before surgery (light, medium, and heavy duty), but this result did reach statistical significance. Alcohol
consumption (more than 6 drinks per week) was the only factor to demonstrate an association with postoperative
restricted work duty and increased rotator cuff repair failure. There is no evidence to support increased infection
rates for rotator cuff repair in Worker’s Compensation patients.
•
•
RECOMMENDED READINGS
Bhatia S, Piasecki DP, Nho SJ, Romeo AA, Cole BJ, Nicholson GP, Boniquit N, Verma NN. Early return to work in
workers' compensation patients after arthroscopic full-thickness rotator cuff repair. Arthroscopy. 2010
Aug;26(8):1027-34. Epub 2010 Jun 3. PubMed PMID: 20678699.
Henn RF 3rd, Kang L, Tashjian RZ, Green A. Patients with workers' compensation claims have worse outcomes after
rotator cuff repair. J Bone Joint Surg Am. 2008 Oct;90(10):2105-13. PubMed PMID: 18829907.
CONFIDENTIAL
55
• A 75-year-old man sustains an anterior
dislocation of his reverse total shoulder
arthroplasty. What activity places the arm in the
position most commonly associated with reverse
total shoulder dislocation?
• 1- Scratching the opposite shoulder
2- Pushing off an ipsilateral chair armrest to assist
in standing up
3- Tying shoelaces on the contralateral foot
4- Brushing hair
CONFIDENTIAL
56
•
PREFERRED RESPONSE: 2- Pushing off an ipsilateral chair armrest to assist in standing up
•
•
DISCUSSION
Proper soft-tissue tension is critical to prevent instability of a reverse total shoulder implanted with the
deltopectoral approach; dislocation of the prosthesis is exceedingly rare if the superior approach is employed. The
arm position implicated in reverse total shoulder instability is extension, adduction, and internal rotation, such as
pushing out of a chair. The other positions described do not involve extension of the shoulder.
•
•
RECOMMENDED READINGS
Gerber C, Pennington SD, Nyffeler RW. Reverse total shoulder arthroplasty. J Am Acad Orthop Surg. 2009
May;17(5):284-95. Review. PubMed PMID: Cheung E, Willis M, Walker M, Clark R, Frankle MA. Complications in
reverse total shoulder arthroplasty. J Am Acad Orthop Surg. 2011 Jul;19(7):439-49. Review. PubMed PMID:
21724923.19411640.
View Abstract at PubMed
Cheung E, Willis M, Walker M, Clark R, Frankle MA. Complications in reverse total shoulder arthroplasty. J Am Acad
Orthop Surg. 2011 Jul;19(7):439-49. Review. PubMed PMID: 21724923.
View Abstract at PubMed
Walch G, Wall B, Mottier F: Complication and revision of the reverse prosthesis: A multicenter study of 457 cases.
In: Walch G, Boileau P, Mole P, Favard L, Levigne C, Sirveaux f, eds. Reverse Shoulder Arthroplasty: Clinical Results,
Complications, Revision. Montpellier, France: Sauramps Médical; 2006:335-352.
•
•
CONFIDENTIAL
57
• When performing an ulnar nerve decompression at the
elbow, the surgeon must be aware of the
• 1- median nerve as it crosses the surgical field 6 cm
proximal to the medial epicondyle.
2- medial antebrachial cutaneous nerve as it crosses
the field 3 cm distal to the medial epicondyle.
3- anterior antebrachial cutaneous nerve as it crosses
the field at the medial epicondyle.
4- posterior antebrachial cutaneous nerve that crosses
the field 2 cm distal to the medial epicondyle.
CONFIDENTIAL
58
•
PREFERRED RESPONSE: 2- medial antebrachial cutaneous nerve as it crosses the field 3 cm distal to
the medial epicondyle.
•
•
DISCUSSION
The medial antebrachial cutaneous and medial brachial cutaneous are nerves that can be injured
during ulnar nerve decompression at the elbow. The medial antebrachial cutaneous nerve crosses
the surgical field at an average of 3.1 cm distal to the medial epicondyle. The medial brachial
cutaneous nerve crosses the field 7 cm proximal to the medial epicondyle and arborizes into 2 to 3
terminal branches. Because the surgical approach involves dissection on the medial side, the
posterior antebrachial cutaneous nerve is distant from the exposure. Although the median nerve
potentially can be located in the deep dissection of a submuscular transposition, it is considered
distant to an in situ decompression.
•
•
RECOMMENDED READINGS
Lowe JB 3rd, Maggi SP, Mackinnon SE. The position of crossing branches of the medial antebrachial
cutaneous nerve during cubital tunnel surgery in humans. Plast Reconstr Surg. 2004 Sep
1;114(3):692-6. PubMed PMID: 15318047.
Chowdhry S, Elston JB, Lefkowitz T, Wilhelmi BJ. Avoiding the medial brachial cutaneous nerve in
brachioplasty: an anatomical study. Eplasty. 2010 Jan 29;10:e16. PubMed PMID: 20165546.
CONFIDENTIAL
59
Figure 33
Figure 33 is the radiograph of a 27-year-old bicyclist who crashes. He has an isolated and closed injury. He is neurovascularly
intact in the upper extremity. The lateral fragment is displaced inferiorly by
1- gravity.
2- the trapezius.
3- the biceps.
4- the pectoralis minor.
CONFIDENTIAL
60
•
PREFERRED RESPONSE: 1- gravity.
•
•
DISCUSSION
Open reduction and internal fixation with a plate and screw construct have been demonstrated to
reduce nonunion rate and improve outcomes compared to sling immobilization for displaced
clavicle fractures. Neurovascular injury and infection risk increase, however, with surgery. In the
upright position, the weight of the extremity inferiorly displaces the lateral segment.
•
•
RECOMMENDED READINGS
Altamimi SA, McKee MD; Canadian Orthopaedic Trauma Society. Nonoperative treatment
compared with plate fixation of displaced midshaft clavicular fractures. Surgical technique. J Bone
Joint Surg Am. 2008 Mar;90 Suppl 2 Pt 1:1-8. PMID: 18310682.
View Abstract at PubMed
Canadian Orthopaedic Trauma Society. Nonoperative treatment compared with plate fixation of
displaced midshaft clavicular fractures. A multicenter, randomized clinical trial. J Bone Joint Surg
Am. 2007 Jan;89(1):1-10. PMID: 17200303.
View Abstract at PubMed
Darouiche RO. Treatment of infections associated with surgical implants. N Engl J Med. 2004 Apr
1;350(14):1422-9. Review. PubMed PMID: 15070792.
•
•
CONFIDENTIAL
61
Figure 34a
Figure 34b
CONFIDENTIAL
62
Placement of the most distal interlocking screw seen in the Figures 34a and
34b radiographs most likely resulted in what motor weakness?
1- Elbow flexion
2- Thumb interphalangeal (IP) extension
3- Index proximal IP flexion
4- Index metacarpophalangeal (MCP) abduction
CONFIDENTIAL
63
•
PREFERRED RESPONSE: 3- Index proximal IP flexion
•
•
DISCUSSION
The most distal locking screw in this intramedullary nail construct was placed from anterior to posterior, passing
through the distal portion of the biceps and brachialis muscle bellies. The median nerve, along with the brachial
artery, is at risk as it lies between these 2 muscles. Malrotation of the nail, producing a more anteromedial starting
point for the screw, can lead to a path that intersects with the nerve. Blunt dissection and soft-tissue protection is
warranted with this screw placement. Median nerve injury would affect innervations of the flexor digitorum
superficialis and profundus to the index finger (among other motors). Although the dissection violates the muscle
belly of these 2 elbow flexors, measurable weakness is not typically seen. The radial nerve has already provided
function to triceps (elbow extension) proximal to this level and lies sufficiently lateral to be more of a concern with
a lateral screw placement (thumb IP extension). The ulnar nerve (index MCP abduction) is further medial at this
level and would be at considerably lower risk than the median.
•
•
RECOMMENDED READINGS
Morrey BF. Anatomy of the elbow joint. In: Morrey BF, Sanchez-Sotelo J, eds. The Elbow and Its Disorders.
Philadelphia, PA: WB Saunders; 2009:11-38.
•
Bohsali KI, Wirth MA. Fractures of the proximal humerus. In: Rockwood CA, Matsen FA, Wirth MA, Lippitt SB, eds.
The Shoulder. Philadelphia, PA: WB Saunders; 2009:315-319.
CONFIDENTIAL
64
• One week after closed reduction of a primary anterior
shoulder dislocation, a 25-year-old athlete should be
counseled that
• 1- recurrence rate is reduced with 4 weeks of
immobilization instead of 2 weeks of immobilization.
2- age at the time of injury is the most consistent risk
factor for recurrent instability.
3- a majority of patients in this age group will elect to
have surgery for recurrent instability.
4- after an in-season return to sports, his likelihood of
choosing surgery after the season is 25%.
CONFIDENTIAL
65
•
PREFERRED RESPONSE: 2- age at the time of injury is the most consistent risk factor for recurrent
instability.
•
•
DISCUSSION
In a study by Sachs and associates, age younger than 25 years at the time of presentation was found
to be the strongest predictor of recurrent instability. In this age group (20- to 29-year-olds), only
14% elected to proceed with surgery. After an in-season return to sports, about 50% of patients in
this same study chose to proceed with surgery after completing the season. Immobilization in a
sling for longer than 2 weeks has no effect on future instability.
•
•
RECOMMENDED READINGS
Sachs RA, Lin D, Stone ML, Paxton E, Kuney M. Can the need for future surgery for acute traumatic
anterior shoulder dislocation be predicted? J Bone Joint Surg Am. 2007 Aug;89(8):1665-74. PubMed
PMID: 17671003.
View Abstract at PubMed
Paterson WH, Throckmorton TW, Koester M, Azar FM, Kuhn JE. Position and duration of
immobilization after primary anterior shoulder dislocation: a systematic review and meta-analysis
of the literature. J Bone Joint Surg Am. 2010 Dec 15;92(18):2924-33. doi: 10.2106/JBJS.J.00631.
Review. PubMed PMID: 21159993.
•
CONFIDENTIAL
66
• CLINICAL SITUATION FOR QUESTIONS 36 THROUGH 39
• A 65-year-old man experienced 6 years of worsening shoulder pain.
Examination demonstrates stiffness and crepitus with range of
motion, but full rotator cuff strength in all planes. Radiographs
show advanced shoulder osteoarthritis, and an MRI scan ordered by
the patient's primary care physician shows an intact rotator cuff.
• What is the most likely glenoid wear pattern seen in glenohumeral
osteoarthritis with an external rotation deficit?
• 1- Posterior wear
2- Anterior wear
3- Central wear
4- Superior wear
CONFIDENTIAL
67
• PREFERRED RESPONSE: 1- Posterior wear
CONFIDENTIAL
68
• Which treatment is most likely to result in longterm pain relief and functional improvement?
• 1- Hemiarthroplasty
2- Hemiarthroplasty with meniscal interposition
3- Total shoulder arthroplasty (TSA)
4- Reverse TSA
• 5-Travel pay to and from Wichita Falls, TX
CONFIDENTIAL
69
• PREFERRED RESPONSE: 3- Total shoulder
arthroplasty (TSA)
CONFIDENTIAL
70
• What risk factor is most predictive of deep
infection following TSA?
• 1- Posttraumatic arthritis
2- Male gender
3- Body mass index higher than 30
4- Diabetes
CONFIDENTIAL
71
• PREFERRED RESPONSE: 2- Male gender
CONFIDENTIAL
72
• A functionally low demand 75-year-old woman
with rheumatoid arthritis and a long history of
oral corticosteroid use sustains a comminuted
intra-articular distal humerus fracture. What is
the best surgical option?
• 1- Open reduction internal fixation (ORIF) with
parallel plates
2- ORIF with orthogonal plates and iliac crest
bone grafting
3- Total elbow arthroplasty (TEA)
4- Closed reduction and percutaneous pinning
CONFIDENTIAL
73
•
•
•
•
PREFERRED RESPONSE: 3- Total elbow arthroplasty (TEA)
•
•
RECOMMENDED READINGS
McKee MD, Veillette CJ, Hall JA, Schemitsch EH, Wild LM, McCormack R, Perey B, Goetz T, Zomar M, Moon K,
Mandel S, Petit S, Guy P, Leung I. A multicenter, prospective, randomized, controlled trial of open reduction-internal fixation versus total elbow arthroplasty for displaced intra-articular distal humeral fractures in elderly
patients. J Shoulder Elbow Surg. 2009 Jan-Feb;18(1):3-12. doi: 10.1016/j.jse.2008.06.005. Epub 2008 Sep 26.
PubMed PMID: 18823799.
Frankle MA, Herscovici D Jr, DiPasquale TG, Vasey MB, Sanders RW. A comparison of open reduction and internal
fixation and primary total elbow arthroplasty in the treatment of intraarticular distal humerus fractures in women
older than age 65. J Orthop Trauma. 2003 Aug;17(7):473-80. PubMed PMID: 12902784.
•
DISCUSSION
TEA is the best surgical option. McKee and associates published a multicenter randomized controlled trial
comparing ORIF to TEA in elderly patients. TEA resulted in better 2-year clinical functional scores and more
predictable outcomes compared to ORIF. TEA was also likely to result in a lower resurgical rate; one-quarter of
patients with fractures randomized to ORIF could not achieve stable fixation. Further, Frankle and associates
reported a comparative study of TEA vs ORIF in 24 elderly women. TEA outcomes were again superior to ORIF at a
minimum of 2 years of followup. TEA was especially useful in patients with comorbidities that compromise bone
stock, including osteoporosis and oral corticosteroid use. Closed reduction and percutaneous pinning studies have
not been published on the adult population.
CONFIDENTIAL
74
• A 67-year-old man with right shoulder osteoarthritis
remains symptomatic despite a course of nonsurgical
treatment. A CT scan of the shoulder before surgery
shows eccentric posterior glenoid wear with 10
degrees of retroversion. What is the appropriate
treatment of this glenoid bone loss?
• 1- Implantation of the glenoid component in 10
degrees of retroversion
2- Hemiarthroplasty
3- Eccentric reaming of glenoid
4- Posterior glenoid bone graft
CONFIDENTIAL
75
•
PREFERRED RESPONSE: 3- Eccentric reaming of glenoid
•
•
DISCUSSION
Total shoulder arthroplasty (TSA) is superior to hemiarthroplasty for primary osteoarthritis. The most common complication of TSA is glenoid
loosening and malposition, which are common causes of glenoid failure. Glenoid malposition decreases the glenohumeral contact area and
subsequently increases contact pressures. Altering the stem version to accommodate glenoid retroversion does not appropriately address soft-tissue
balancing. A retroversion of less than 12 degrees to 15 degrees can be corrected with eccentric reaming without excessively compromising glenoid
bone stock and risking glenoid vault penetration by the glenoid component. Posterior glenoid bone grafting may be considered for glenoid
retroversion exceeding 15 degrees.
•
•
RECOMMENDED READINGS
Levine WN, Fischer CR, Nguyen D, Flatow EL, Ahmad CS, Bigliani LU. Long-Term Follow-up of Shoulder Hemiarthroplasty for Glenohumeral
Osteoarthritis. J Bone Joint Surg Am. 2012 Nov 21;94(22):e1641-7. doi: 10.2106/JBJS.K.00603. PubMed PMID: 23172331.
Edwards TB, Kadakia NR, Boulahia A, Kempf JF, Boileau P, Némoz C, Walch G. A comparison of hemiarthroplasty and total shoulder arthroplasty in the
treatment of primary glenohumeral osteoarthritis: results of a multicenter study. J Shoulder Elbow Surg. 2003 May-Jun;12(3):207-13. PubMed PMID:
12851570.
Sears BW, Johnston PS, Ramsey ML, Williams GR. Glenoid bone loss in primary total shoulder arthroplasty: evaluation and management. J Am Acad
Orthop Surg. 2012 Sep;20(9):604-13. doi: 10.5435/JAAOS-20-09-604. Review. PubMed PMID: 22941803.
Shapiro TA, McGarry MH, Gupta R, Lee YS, Lee TQ. Biomechanical effects of glenoid retroversion in total shoulder arthroplasty. J Shoulder Elbow
Surg. 2007 May-Jun;16(3 Suppl):S90-5. Epub 2006 Dec 12. PubMed PMID: 17169588.
Nowak DD, Bahu MJ, Gardner TR, Dyrszka MD, Levine WN, Bigliani LU, Ahmad CS. Simulation of surgical glenoid resurfacing using three-dimensional
computed tomography of the arthritic glenohumeral joint: the amount of glenoid retroversion that can be corrected. J Shoulder Elbow Surg. 2009
Sep-Oct;18(5):680-8. doi: 10.1016/j.jse.2009.03.019. Epub 2009 May 31. PubMed PMID: 19487133.
•
CONFIDENTIAL
76
Figure 42
Above is the MRI scan of a 52-year-old active Veteran who fell from a ladder 6 weeks ago and sustained an isolated
glenohumeral dislocation that was reduced in the emergency department. He wore his sling for about 2 weeks with his pack
of cigarettes tucked inside, and arrived at your clinic today after referral by his primary care doctor for a tear in his “rotary
cup”. Examination reveals sensation intact throughout his hand, forearm, and shoulder girdle. Belly press examination and
lift off test are abnormal. He has tenderness to palpation on the anterior shoulder and a painful Speed test. Rotator cuff
repair associated with tenotomy of the indicated structure will result in what condition when compared to tenodesis of the
same structure?
1- Decreased strength in forward elevation
2- Increased external rotation
3- Cosmetic deformity
4- Earlier fatigability with pronation
CONFIDENTIAL
77
•
PREFERRED RESPONSE: 3- Cosmetic deformity
•
•
DISCUSSION
Patients with subluxation of the biceps tendon and full-thickness tears of the
subscapularis require treatment of the biceps tendon. Studies have shown there is
increased likelihood for cosmetic “Popeye” deformity after tenotomy when
compared to tenodesis, but there is little or no functional deficit associated with
tenotomy. In other studies, there have been findings of supination strength
reduction in patients with tenotomy compared to those with tenodesis.
•
•
RECOMMENDED READINGS
Hsu AR, Ghodadra NS, Provencher MT, Lewis PB, Bach BR. Biceps tenotomy versus
tenodesis: a review of clinical outcomes and biomechanical results. J Shoulder
Elbow Surg. 2011 Mar;20(2):326-32. Epub 2010 Nov 4. Review. PMID: 21051241.
Slenker NR, Lawson K, Ciccotti MG, Dodson CC, Cohen SB. Biceps tenotomy versus
tenodesis: clinical outcomes. Arthroscopy. 2012 Apr;28(4):576-82. Epub 2012 Jan
28. Review. PMID: 22284407.
CONFIDENTIAL
78
CONFIDENTIAL
79
• CLINICAL SITUATION FOR QUESTIONS 44 AND 45
• A 19-year-old hockey player returns home from college
over holiday break and experiences multiple recurrent
dislocations only 1 year after an arthroscopic
stabilization.
• Question 44
• What is the preferred test to evaluate this patient?
• 1- Electromyography
2- MRI scan with intravenous contrast
3 Bone scan
4- CT arthrogram
CONFIDENTIAL
80
• PREFERRED RESPONSE: 4- CT arthrogram
CONFIDENTIAL
81
• Question 45
• The treating physician opted to perform a Latarjet
coracoid transfer. What is the primary mechanism
of stability?
• 1- Capsular reinforcement by the coracoacromial
ligament
2- Dynamic sling created by the conjoint tendon
3- Increased glenoid depth
4- Subscapularis tenodesis
CONFIDENTIAL
82
•
PREFERRED RESPONSE: 3- Increased glenoid depth
•
•
DISCUSSION
Because bone loss is likely the biggest risk factor for this patient’s recurrence, a CT arthrogram will provide the
most accurate representation of the defect. An arthrogram enhances the ability to evaluate the capsule and
ligamentous attachments that may have a role in recurrent instability. Collision athletes may have a lower
recurrence rate with open surgery. The Latarjet coracoid transfer achieves its primary stability through the
increased glenoid surface area that the bone block provides.
•
•
RECOMMENDED READINGS
Schmid SL, Farshad M, Catanzaro S, Gerber C. The Latarjet procedure for the treatment of recurrence of anterior
instability of the shoulder after operative repair: a retrospective case series of forty-nine consecutive patients. J
Bone Joint Surg Am. 2012 Jun 6;94(11):e75. doi: 10.2106/JBJS.K.00380. PubMed PMID: 22637215.
Allain J, Goutallier D, Glorion C. Long-term results of the Latarjet procedure for the treatment of anterior instability
of the shoulder. J Bone Joint Surg Am. 1998 Jun;80(6):841-52. PubMed PMID: 9655102.
Ghodadra N, Gupta A, Romeo AA, Bach BR Jr, Verma N, Shewman E, Goldstein J, Provencher MT. Normalization of
glenohumeral articular contact pressures after Latarjet or iliac crest bone-grafting. J Bone Joint Surg Am. 2010
Jun;92(6):1478-89. doi: 10.2106/JBJS.I.00220. PubMed PMID: 20516324.
Yamamoto N, Muraki T, Sperling JW, Steinmann SP, Cofield RH, Itoi E, An KN. Stabilizing mechanism in bonegrafting of a large glenoid defect. J Bone Joint Surg Am. 2010 Sep 1;92(11):2059-66. doi: 10.2106/JBJS.I.00261.
PubMed PMID: 20810855.
Thomazeau H, Courage O, Barth J, Pélégri C, Charousset C, Lespagnol F, Nourissat G, Audebert S, Guillo S, Toussaint
B, Lafosse L, Bradel J, Veillard D, Boileau P; French Arthroscopy Society. Can we improve the indication for Bankart
arthroscopic repair? A preliminary clinical study using the ISIS score. Orthop Traumatol Surg Res. 2010 Dec;96(8
Suppl):S77-83. doi: 10.1016/j.otsr.2010.09.007. Epub 2010 Oct 28. PubMed PMID: 21035419.
•
CONFIDENTIAL
83
• A 45-year-old woman has a 3-month history of left shoulder pain. She has
tried 2 months of physical therapy focused on rotator cuff strengthening
without experiencing relief. A subacromial corticosteroid injection fails to
provide lasting relief. Examination reveals no atrophy or winging. She has
anterior and posterior shoulder tenderness, full symmetric forward
elevation and abduction, and pain with maximal passive forward
elevation. She has pain with internal rotation in 90 degrees of forward
elevation. She has an increased distance between the antecubital fossa
and coracoid process with cross chest adduction compared to the
contralateral side. No weakness is appreciated. Radiographs reveal a type
II acromion. What is the best next step?
• 1- Posterior capsular stretching
2- Arthroscopic subacromial decompression
3- Diagnostic acromioclavicular (AC) joint injection
4- MRI scan
• 5-Tell her to soldier on.
CONFIDENTIAL
84
•
PREFERRED RESPONSE: 1- Posterior capsular stretching
•
•
DISCUSSION
This patient’s examination is consistent with posterior capsular tightness, which can mimic
impingement. Four weeks of posterior capsular stretches will likely improve motion and pain.
Surgical treatment should be considered only after failure of a dedicated stretching program. An AC
joint injection would help differentiate this condition from AC joint arthritis, but in light of the
radiographic findings, arthritis is unlikely. An MRI scan likely will not change the initial treatment at
this point.
•
•
RECOMMENDED READINGS
Bach HG, Goldberg BA. Posterior capsular contracture of the shoulder. J Am Acad Orthop Surg. 2006
May;14(5):265-77. Review. PubMed PMID: 16675620.
Papadonikolakis A, McKenna M, Warme W, Martin BI, Matsen FA 3rd. Published evidence relevant
to the diagnosis of impingement syndrome of the shoulder. J Bone Joint Surg Am. 2011 Oct
5;93(19):1827-32. doi: 10.2106/JBJS.J.01748. Review. PubMed PMID: 22005869.
CONFIDENTIAL
85
• CLINICAL SITUATION FOR QUESTIONS 47 THROUGH 49
• A 13-year-old pitcher reports the immediate onset of
medial elbow pain after throwing a pitch. Upon
examination, the patient is tender to palpation at the
medial epicondyle and has pain and instability with
valgus testing of the elbow.
• Question 47
• Which is the most appropriate diagnostic test?
• 1- MRI arthrogram
2- CT scan with 3-dimensional reconstructions
3- Plain radiographs of both elbows
4- Ultrasound
CONFIDENTIAL
86
• PREFERRED RESPONSE: 3- Plain radiographs of
both elbows
CONFIDENTIAL
87
• Question 48
• Why was your response for question 47 the
most appropriate test for this patient?
• 1- To evaluate for apophyseal injury
2- To evaluate for osteochondral defect
3- To evaluate for hematoma
4- To evaluate for valgus overload injury
CONFIDENTIAL
88
• PREFERRED RESPONSE: 1- To evaluate for
apophyseal injury
CONFIDENTIAL
89
• Question 49
• If the patient were a college pitcher with a
similar presentation and examination, what
structure would most likely be injured?
• 1- Ulnar collateral ligament
2- Pronator teres
3- Ligament of Struthers
4- Lateral collateral ligament
CONFIDENTIAL
90
•
PREFERRED RESPONSE: 1- Ulnar collateral ligament
•
•
DISCUSSION
The patient has an acute avulsion fracture of the medial epicondyle, which can occur in response to
the valgus load placed on the elbow while throwing. Diagnosis is confirmed by radiograph, with
comparison views of the uninjured elbow to evaluate for physeal closure vs injury. In older pitchers,
the ulnar collateral ligament fails rather than the bone of the medial epicondyle. Advanced imaging
may be necessary to confirm the diagnosis of an ulnar collateral ligament injury and/or bony injury.
•
•
•
RECOMMENDED READINGS
Osbahr DC, Chalmers PN, Frank JS, Williams RJ 3rd, Widmann RF, Green DW. Acute, avulsion
fractures of the medial epicondyle while throwing in youth baseball players: a variant of Little
League elbow. J Shoulder Elbow Surg. 2010 Oct;19(7):951-7. doi: 10.1016/j.jse.2010.04.038. Epub
2010 Aug 5. PubMed PMID: 20688542.
Gottschalk HP, Eisner E, Hosalkar HS. Medial epicondyle fractures in the pediatric population. J Am
Acad Orthop Surg. 2012 Apr;20(4):223-32. doi: 10.5435/JAAOS-20-04-223. Review. PubMed PMID:
22474092.
CONFIDENTIAL
91
• A 65-year-old woman has electrodiagnostic findings of
ulnar nerve entrapment at the elbow. You counsel the
patient that
• 1- medial epicondylectomy and submuscular transposition
is the preferred treatment.
2- arthroscopic decompression is associated with lower
complication rates compared to open treatment.
3- simple decompression can be as effective as ulnar nerve
transposition.
4- duration of symptoms is the most important predictor of
outcome.
• 5-It will take 4-6 hours for SPS to sterilize the instruments
before any surgical intervention can be considered.
CONFIDENTIAL
92
•
PREFERRED RESPONSE: 3- simple decompression can be as effective as ulnar nerve transposition.
•
•
DISCUSSION
Multiple studies have demonstrated that simple ulnar nerve decompression is as effective as subcutaneous transposition for most symptomatic ulnar
neuropathy. Ulnar nerve instability before or after decompression may best be treated by transposition rather than simple decompression.
Submuscular transposition with or without medial epicondylectomy may be best reserved for revision surgery or patients who are exceedingly thin.
Arthroscopic nerve decompression has been reported with arthroscopic treatment of elbow arthritis, but is associated with a higher complication
and revision rate than the standard techniques. Adequate nerve decompression, rather than duration of symptoms, is the most important predictor
of outcome.
•
•
RECOMMENDED READINGS
Caliandro P, La Torre G, Padua R, Giannini F, Padua L. Treatment for ulnar neuropathy at the elbow. Cochrane Database Syst Rev. 2012 Jul
11;7:CD006839. doi: 10.1002/14651858.CD006839.pub3. Review. PubMed PMID: 22786500.
Zlowodzki M, Chan S, Bhandari M, Kalliainen L, Schubert W. Anterior transposition compared with simple decompression for treatment of cubital
tunnel syndrome. A meta-analysis of randomized, controlled trials. J Bone Joint Surg Am. 2007 Dec;89(12):2591-8. PubMed PMID: 18056489.
Macadam SA, Gandhi R, Bezuhly M, Lefaivre KA. Simple decompression versus anterior subcutaneous and submuscular transposition of the ulnar
nerve for cubital tunnel syndrome: a meta-analysis. J Hand Surg Am. 2008 Oct;33(8):1314.e1-12. doi: 10.1016/j.jhsa.2008.03.006. Review. PubMed
PMID: 18929194.
Gellman H. Compression of the ulnar nerve at the elbow: cubital tunnel syndrome. Instr Course Lect. 2008;57:187-97. Review. PubMed PMID:
18399580.
Kovachevich R, Steinmann SP. Arthroscopic ulnar nerve decompression in the setting of elbow osteoarthritis. J Hand Surg Am. 2012 Apr;37(4):663-8.
doi: 10.1016/j.jhsa.2012.01.003. Epub 2012 Mar 3. PubMed PMID: 22386545.
Shi Q, MacDermid JC, Santaguida PL, Kyu HH. Predictors of surgical outcomes following anterior transposition of ulnar nerve for cubital tunnel
syndrome: a systematic review. J Hand Surg Am. 2011 Dec;36(12):1996-2001.e1-6. doi: 10.1016/j.jhsa.2011.09.024. Review. PubMed PMID:
22123047.
CONFIDENTIAL
93
• A 15-year-old girl has experienced 6 months of increasing dominant
shoulder pain while playing volleyball. Her pain is so significant that
she can no longer compete. Examination demonstrates 190 degrees
of forward elevation, 110 degrees of external rotation at the side,
and internal rotation up the back to T2 bilaterally. She also has 15
degrees of bilateral elbow hyperextension. Load and shift testing
demonstrates pain with anterior and posterior drawer tests. She
has a large sulcus on examination that causes pain during testing.
Forward elevation and external rotation strength testing shows 4/5
strength. There is no scapular winging and radiograph findings are
normal. What is the best next step?
• 1- Physical therapy for rotator cuff strengthening
2- Subacromial corticosteroid injection
3- MRI arthrogram
4- Arthroscopic stabilization
CONFIDENTIAL
94
•
PREFERRED RESPONSE: 1- Physical therapy for rotator cuff strengthening
•
•
DISCUSSION
This patient has multidirectional instability as evidenced by her hyperlaxity and excessive range of
motion. Patients with pain after activities often have weak rotator cuff musculature and improve
with strengthening of the rotator cuff and proprioceptive retraining. Subacromial injection likely
cannot help this patient because it will not treat her underlying motor weakness in the rotator cuff
or her dyskinesia. MRI arthrogram is not indicated unless she fails nonsurgical treatment.
Arthroscopic stabilization also would be reserved for patients who fail nonsurgical treatment.
•
•
RECOMMENDED READINGS
Gaskill TR, Taylor DC, Millett PJ. Management of multidirectional instability of the shoulder. J Am
Acad Orthop Surg. 2011 Dec;19(12):758-67. Review. PMID: 22134208.
Jacobson ME, Riggenbach M, Wooldridge AN, Bishop JY. Open capsular shift and arthroscopic
capsular plication for treatment of multidirectional instability. Arthroscopy. 2012 Jul;28(7):1010-7.
Review. PMID: 22365265.
•
CONFIDENTIAL
95
Figure 55a
Figure 55b
Figures 55a and 55b are the radiographs of a 64-year-old woman with a history significant for rheumatoid arthritis who has the
chief complaint of right elbow pain. She has been treated with tumor necrosis factor-alpha inhibitors and oral corticosteroids for
several years. The patient experiences severe global elbow pain and crepitus. What process primarily is responsible for joint
destruction in rheumatoid arthritis?
1- Traumatic insult resulting in complement activation
2- Mutation in the rheumatoid factor gene
3- Osteoblast paracrine signaling resulting in proteolytic collagen degradation
4- Inflammation resulting in a hyperplastic synovial joint lining
CONFIDENTIAL
96
•
•
•
•
PREFERRED RESPONSE: 4- Inflammation resulting in a hyperplastic synovial joint lining
•
•
RECOMMENDED READINGS
Chen AL, Joseph TN, Zuckerman JD. Rheumatoid arthritis of the shoulder. J Am Acad Orthop Surg.
2003 Jan-Feb;11(1):12-24. Review. PubMed PMID: 12699368.
Larsen A, Dale K, Eek M. Radiographic evaluation of rheumatoid arthritis and related conditions by
standard reference films. Acta Radiol Diagn (Stockh). 1977 Jul;18(4):481-91. PubMed PMID:
920239.
Gill DR, Morrey BF. The Coonrad-Morrey total elbow arthroplasty in patients who have rheumatoid
arthritis. A ten to fifteen-year follow-up study. J Bone Joint Surg Am. 1998 Sep;80(9):1327-35.
PubMed PMID: 9759818.
DISCUSSION
Rheumatoid arthritis is a systemic inflammatory disorder marked by erosive arthritis in multiple
joints. Elbow involvement is common. The pathologic lesion in rheumatoid arthritis is pannus, a
hyperplastic synovial proliferation that ultimately results in proteoglycan and collagen digestion.
Rheumatoid factor mutations, traumatic insults resulting in complement activation and osteoblast
paracrine signaling, are not involved in the pathologic process.
CONFIDENTIAL
97
• A 35-year-old man fell off of a roof and sustained an extraarticular supracondylar elbow fracture. He had normal
sensation in all fingers after the injury and before
undergoing surgery to repair the fracture. The ulnar nerve
was not transposed, but it was inspected prior to wound
closure. Ten days after surgery, the patient has numbness in
his small finger and is unable to cross his fingers. His elbow
range of motion is between 40 degrees and 100 degrees.
What is the next appropriate treatment step?
• 1- Elbow splint at 40 degrees at night for 6 weeks
2- Electromyography (EMG)
3- Exploration of the ulnar nerve and transposition
4- Observation
CONFIDENTIAL
98
•
PREFERRED RESPONSE: 4- Observation
•
•
DISCUSSION
This patient has an early postsurgical ulnar nerve palsy. The causes of this injury
are laceration of the nerve during surgery, entrapment of the nerve in the fracture
or hardware, or traction injury during surgery. If the orthopaedic surgeon is sure
that the nerve was not lacerated at the end of the case or entrapped in the
hardware, then the nerve is probably intact and will recover. Observation is the
best treatment in this case because the nerve was checked before wound closure.
Elbow splinting has not been shown to help with postsurgical nerve recovery. EMG
findings may not be accurate this early in the injury.
•
•
•
RECOMMENDED READINGS
Shin R, Ring D. The ulnar nerve in elbow trauma. J Bone Joint Surg Am. 2007
May;89(5):1108-16. Review. PubMed PMID: 17473151.
Faierman E, Wang J, Jupiter JB. Secondary ulnar nerve palsy in adults after elbow
trauma: a report of two cases. J Hand Surg Am. 2001 Jul;26(4):675-8. PubMed
PMID: 11466643.
CONFIDENTIAL
99
• A 54-year-old pipefitter falls from a ladder at work and
dislocates his nondominant shoulder. His MRI scan
shows supraspinatus and infraspinatus tears with
retraction to the glenoid. He cannot actively raise his
arm away from his side. He denies prior shoulder
symptoms before his fall. Three weeks of physical
therapy have failed to improve his function. You and
the patient decide to proceed with surgical repair.
Which is a risk factor for a poor outcome?
• 1- The patient’s age
2- The patient’s gender
3- Work-related injury
4- Acute nature of the tear
CONFIDENTIAL
100
•
PREFERRED RESPONSE: 3- Work-related injury
•
•
DISCUSSION
Several studies have demonstrated that patients with work-related injuries do not
do as well as those whose injuries are not work-related after repair of the rotator
cuff. This patient’s age and gender are not negative prognostic indicators. The
acute nature of the tear does not lead to an inferior outcome.
•
•
RECOMMENDED READINGS
Kemp KA, Sheps DM, Luciak-Corea C, Styles-Tripp F, Buckingham J, Beaupre LA.
Systematic review of rotator cuff tears in workers' compensation patients. Occup
Med (Lond). 2011 Dec;61(8):556-62. Epub 2011 Oct 19. Review. PMID: 22016341.
Bhatia S, Piasecki DP, Nho SJ, Romeo AA, Cole BJ, Nicholson GP, Boniquit N, Verma
NN. Early return to work in workers' compensation patients after arthroscopic fullthickness rotator cuff repair. Arthroscopy. 2010 Aug;26(8):1027-34. Epub 2010 Jun
3. PMID: 20678699.
CONFIDENTIAL
101
•
•
RESPONSES FOR QUESTIONS 64 THROUGH 68
1- Internal rotation stretching, core stability exercises, and scapular stabilization exercises
2- Arthroscopic debridement
3- Arthroscopic debridement with subacromial decompression
4- Arthroscopic transtendinous repair
5- Arthroscopic tear completion and repair
•
•
•
Please match the interventions above to the scenarios below.
Question 64
A 23-year-old Division 1 baseball pitcher is experiencing worsening pain despite completion of an
extensive, but unsuccessful, sport-specific physical therapy regimen. An MRI scan shows articular
surface tearing of the rotator cuff and internal impingement on abduction external rotation views.
1- Internal rotation stretching, core stability exercises, and scapular stabilization exercises
2- Arthroscopic debridement
3- Arthroscopic debridement with subacromial decompression
4- Arthroscopic transtendinous repair
5- Arthroscopic tear completion and repair
•
CONFIDENTIAL
102
• PREFERRED RESPONSE: 2- Arthroscopic
debridement
CONFIDENTIAL
103
• Question 65
• A 55-year-old woman with a bursal-sided tear
less than 20% thickness and lateral acromial
impingement has failed physical therapy.
• 1- Internal rotation stretching, core stability
exercises, and scapular stabilization exercises
2- Arthroscopic debridement
3- Arthroscopic debridement with subacromial
decompression
4- Arthroscopic transtendinous repair
5- Arthroscopic tear completion and repair
CONFIDENTIAL
104
• PREFERRED RESPONSE: 3- Arthroscopic
debridement with subacromial decompression
CONFIDENTIAL
105
• Question 68
• A 65-year-old man who has failed nonsurgical
treatment demonstrates a partial-thickness
supraspinatus tendon tear of 70% thickness.
• 1- Internal rotation stretching, core stability
exercises, and scapular stabilization exercises
2- Arthroscopic debridement
3- Arthroscopic debridement with subacromial
decompression
4- Arthroscopic transtendinous repair
5- Arthroscopic tear completion and repair
CONFIDENTIAL
106
•
PREFERRED RESPONSE: 5- Arthroscopic tear completion and repair
•
•
DISCUSSION
For the young athlete, a careful examination of scapular mechanics, core stability, and internal rotation deficits is important to diagnose, prevent,
and treat a painful shoulder. Correction of the capsular contracture and core imbalance often is enough to alleviate symptoms. If nonsurgical
management fails, MRI scan findings and arthroscopic examination often show partial-thickness tearing of the articular surface of the supraspinatus
or infraspinatus with or without associated internal impingement or aberrant contact with the posterior superior labrum and glenoid. The prevalence
may be as high as 20% to 40% in the overhead athletic population, likely resulting from repetitive microtrauma. Degenerative tears often become
symptomatic with an increase in size; the mechanical linkage between the supraspinatus and infraspinatus makes it likely that the remaining fibers
bear more strain as tear size increases. Simple acromioplasty may be successful only in cases of definitive contact between the acromion and
supraspinatus. Transtendinous repairs are effective treatment for partial-thickness tears of the supraspinatus, but they may have a slower functional
recovery and a higher rate of stiffness than excision and repair; this may be attributable, in part, to the natural overlap of the infraspinatus tendon
over the supraspinatus tendon. An “all-inside” technique may be preferable in younger patients because it is possible to reinsert only the surface
fibers that are torn, avoiding constraint of the superficial, bursal fibers. For tears of more than 50% thickness, completing the tear to excise the
remaining degenerative fibers may be the preferred treatment.
•
•
RECOMMENDED READINGS
Ide J, Maeda S, Takagi K. Arthroscopic transtendon repair of partial-thickness articular-side tears of the rotator cuff: anatomical and clinical study. Am
J Sports Med. 2005 Nov;33(11):1672-9. Epub 2005 Aug 10. PubMed PMID: 16093533.
Yang S, Park HS, Flores S, Levin SD, Makhsous M, Lin F, Koh J, Nuber G, Zhang LQ. Biomechanical analysis of bursal-sided partial thickness rotator cuff
tears. J Shoulder Elbow Surg. 2009 May-Jun;18(3):379-85. doi: 10.1016/j.jse.2008.12.011. Epub 2009 Mar 9. PubMed PMID: 19269860.
Finnan RP, Crosby LA. Partial-thickness rotator cuff tears. J Shoulder Elbow Surg. 2010 Jun;19(4):609-16. doi: 10.1016/j.jse.2009.10.017. Epub 2010
Feb 19. Review. PubMed PMID: 20171904.
Kamath G, Galatz LM, Keener JD, Teefey S, Middleton W, Yamaguchi K. Tendon integrity and functional outcome after arthroscopic repair of highgrade partial-thickness supraspinatus tears. J Bone Joint Surg Am. 2009 May;91(5):1055-62. doi: 10.2106/JBJS.G.00118. Erratum in: J Bone Joint Surg
Am. 2009 Aug;91(8):1995. PubMed PMID: 19411453.
•
CONFIDENTIAL
107
• A 17-year-old left-hand-dominant gymnast has a 10-week history of
gradually progressive right shoulder pain. She reports the onset of
pain to be associated with an increase in her training regimen while
preparing for an upcoming regional competition, and denies any
specific trauma to her shoulder. Examination reveals end-range
discomfort, but normal active and passive range of motion. Her
periscapular musculature strength is normal, but she demonstrates
mild medial scapular winging with arm elevation. She has 20
degrees’ elbow recurvatum, a positive sulcus examination, and can
hyperextend the metacarpophalangeal joint of her index finger to
105 degrees. What is the most appropriate initial treatment?
• 1- Physical therapy referral for rotator cuff and periscapular
conditioning
2- Electromyography
3- Subacromial injection
4- Arthroscopic capsular plication
CONFIDENTIAL
108
•
PREFERRED RESPONSE: 1- Physical therapy referral for rotator cuff and periscapular conditioning
•
•
DISCUSSION
This patient has shoulder pain and an underlying diagnosis of multidirectional glenohumeral laxity. There remains
controversy as to whether athletes with features of generalized laxity are at increased risk for shoulder pain. This
athlete’s presentation is typical in that the onset was atraumatic and associated with an increase in her training
regimen. The most appropriate treatment step is rest from competition and institution of a dedicated physical
therapeutic exercise program that emphasizes rotator cuff and periscapular strengthening, with a focus on the
serratus anterior. Electromyography would not be helpful because the patient’s isolated periscapular motor
function is intact and she demonstrates a typical pattern of acquired scapular dyskinesis seen in painful overhead
athletic shoulder. Subacromial injection therapy is not indicated in the setting of multidirectional instability.
Arthroscopic surgical options are considered as a final treatment intervention after nonsurgical measures have
failed, and appropriate imaging, such as MRI scan, has been obtained to determine the presence or absence of
significant structural abnormalities.
•
•
RECOMMENDED READINGS
Bak K. The practical management of swimmer's painful shoulder: etiology, diagnosis, and treatment. Clin J Sport
Med. 2010 Sep;20(5):386-90. doi: 10.1097/JSM.0b013e3181f205fa. PubMed PMID: 20818199.
Madsen PH, Bak K, Jensen S, Welter U. Training induces scapular dyskinesis in pain-free competitive swimmers: a
reliability and observational study. Clin J Sport Med. 2011 Mar;21(2):109-13. doi:
10.1097/JSM.0b013e3182041de0. PubMed PMID: 21358500
CONFIDENTIAL
109
• CLINICAL SITUATION FOR QUESTIONS 70 THROUGH 72
• A 17-year-old high school football player sustains a traumatic anterior
shoulder dislocation resulting in a small bony Bankart lesion and small HillSachs lesion. The patient undergoes an arthroscopic Bankart repair with
incorporation of the bone fragment and returns to play football the
following year. He has a recurrent dislocation at football practice, but
decides to finish the football season before considering additional
treatment. He sustains 9 additional dislocations, with the last dislocation
occurring while sleeping.
• Question 70
• What diagnostic test is best when planning revision surgery?
• 1- CT scan with 3-dimensional (3-D) reconstructions
2- Ultrasound
3- MRI scan
4- Arthrogram
CONFIDENTIAL
110
• PREFERRED RESPONSE: 1- CT scan with 3dimensional (3-D) reconstructions
CONFIDENTIAL
111
• Question 71
• The patient has eroded one-third of the inferior
glenoid surface area. What is the most appropriate
treatment?
• 1- A hug
• 2-Revision arthroscopic Bankart repair with capsular
shift
3- Open Bankart repair with capsular shift
4- Repair of infraspinatus tendon into the Hill-Sachs
defect (remplissage procedure)
5- Coracoid transfer to the glenoid (Latarjet procedure)
CONFIDENTIAL
112
• PREFERRED RESPONSE: 4- Coracoid transfer to
the glenoid (Latarjet procedure)
CONFIDENTIAL
113
• Which patients are clinically most dissatisfied
after revision instability surgery?
• 1- Patients with pain before surgery
2- Patients younger than 25 years of age
3- Patients older than 55 years of age
4- Recreational athletes
CONFIDENTIAL
114
•
PREFERRED RESPONSE: 1- Patients with pain before surgery
•
•
DISCUSSION
A failed bony Bankart repair with multiple dislocations can further erode the anteroinferior glenoid, changing the
sagittal morphology of the glenoid into an “inverted pear.” Quantitative bone loss is best evaluated by CT scan
with 3-D reconstructions and subtraction of the humeral head. MRI and ultrasound can assist in evaluating softtissue injury, but they are not as helpful in determining bone loss compared to a CT scan. An arthrogram alone is
not sufficient to evaluate bone loss. Bone loss exceeding 30% necessitates glenoid augmentation with either a
Latarjet procedure or iliac crest bone grafting. A revision arthroscopic or open Bankart repair with capsular shift or
remplissage do not address bone loss. The Latarjet procedure can effectively restore stability with glenoid bone
loss and after failed stabilizing procedures. Patients with pain before surgery are more likely to have pain after
surgery. Age and activity level are lesser influences on satisfaction.
•
•
RECOMMENDED READINGS
Schmid SL, Farshad M, Catanzaro S, Gerber C. The Latarjet procedure for the treatment of recurrence of anterior
instability of the shoulder after operative repair: a retrospective case series of forty-nine consecutive patients. J
Bone Joint Surg Am. 2012 Jun 6;94(11):e75. doi: 10.2106/JBJS.K.00380. PubMed PMID: 22637215.
Piasecki DP, Verma NN, Romeo AA, Levine WN, Bach BR Jr, Provencher MT. Glenoid bone deficiency in recurrent
anterior shoulder instability: diagnosis and management. J Am Acad Orthop Surg. 2009 Aug;17(8):482-93. Review.
PubMed PMID: 19652030.
Burkhart SS, De Beer JF. Traumatic glenohumeral bone defects and their relationship to failure of arthroscopic
Bankart repairs: significance of the inverted-pear glenoid and the humeral engaging Hill-Sachs lesion. Arthroscopy.
2000 Oct;16(7):677-94. PubMed PMID: 11027751.
CONFIDENTIAL
115
Figure 73
Question 73 of 105
Figure 73 is the radiograph of a 78-year-old man who has had 8 months of gradually progressive right shoulder pain. He
temporarily responds to a corticosteroid injection administered by his primary physician, but his symptoms quickly return.
He reports significant interference with activities of daily living and recreational activities. Examination demonstrates
active range of motion to 90 degrees’ forward elevation, 20 degrees’ external rotation at the side, and 50 degrees’ in the
abducted position, with pain at end range. The most appropriate next treatment step is
1- Arthroscopic glenohumeral debridement, synovectomy, and biceps tenotomy
2- Total shoulder arthroplasty (TSA)
3- Reverse TSA (rTSA)
4- Humeral head arthroplasty without glenoid resurfacing
CONFIDENTIAL
116
•
PREFERRED RESPONSE: 3- Reverse TSA (rTSA)
•
•
DISCUSSION
This patient presents with a clinical history, examination, and imaging consistent with end-stage rotator cuff tear
arthropathy. Recommended treatment is rTSA. With significantly limited function and advanced radiographic
changes, arthroscopic intervention is unlikely to provide significant clinical benefit. TSA, with or without rotator
cuff repair, likely would lead to early mechanical failure of the glenoid component (edge loading, or “rocking
horse” glenoid). Hemiarthroplasty was previously the recommended treatment option, prior to the reintroduction
of the reverse implant. However, current data suggest reverse arthroplasty provides a more predictable outcome
(pain relief, improved function) and less need for surgical revision.
•
•
RECOMMENDED READINGS
Cuff D, Pupello D, Virani N, Levy J, Frankle M. Reverse shoulder arthroplasty for the treatment of rotator cuff
deficiency. J Bone Joint Surg Am. 2008 Jun;90(6):1244-51. doi: 10.2106/JBJS.G.00775. PubMed PMID: 18519317.
Drake GN, O'Connor DP, Edwards TB. Indications for reverse total shoulder arthroplasty in rotator cuff disease. Clin
Orthop Relat Res. 2010 Jun;468(6):1526-33. doi: 10.1007/s11999-009-1188-9. Review. PubMed PMID: 20049573.
Leung B, Horodyski M, Struk AM, Wright TW. Functional outcome of hemiarthroplasty compared with reverse total
shoulder arthroplasty in the treatment of rotator cuff tear arthropathy. J Shoulder Elbow Surg. 2012
Mar;21(3):319-23. doi: 10.1016/j.jse.2011.05.023. Epub 2011 Aug 26. PubMed PMID: 21872496.
•
CONFIDENTIAL
117
• A 33-year old man sustains a posterior elbow
dislocation after a fall while intoxicated. Attempts at
closed reduction result in recurrent instability. What is
the most common ligamentous injury found at the
time of surgical stabilization?
• 1- Midsubstance tear of the lateral ulnar collateral
ligament
2- Proximal avulsion of the ulnar collateral ligament
3- Proximal avulsion of the lateral ulnar collateral
ligament
4- Distal bony avulsion of the ulnar collateral ligament
from the sublime tubercle
CONFIDENTIAL
118
•
PREFERRED RESPONSE: 3- Proximal avulsion of the lateral ulnar collateral ligament
•
•
DISCUSSION
Classic posterior elbow dislocations result from a posterolateral rotatory mechanism, whereby the
hand is fixed (typically on the ground) while the weight of the body creates a valgus and external
rotation moment on the elbow. This results first in tearing of the lateral collateral ligament that
proceeds medially through the anterior and posterior joint capsules, ending with potential
involvement of the ulnar collateral ligament (but this is not universal). McKee and associates
assessed the lateral soft-tissue injury pattern of elbow dislocations with and without associated
fractures at the time of surgery. Injury to the lateral collateral ligament complex was seen in every
case, with avulsion from the distal humerus as the most common finding. Midsubstance tears,
proximal avulsions, and distal bony avulsions of the ulnar collateral ligament are less common.
•
•
RECOMMENDED READINGS
McKee MD, Schemitsch EH, Sala MJ, O'driscoll SW. The pathoanatomy of lateral ligamentous
disruption in complex elbow instability. J Shoulder Elbow Surg. 2003 Jul-Aug;12(4):391-6. PubMed
PMID: 12934037.
O'Driscoll SW, Morrey BF, Korinek S, An KN. Elbow subluxation and dislocation. A spectrum of
instability. Clin Orthop Relat Res. 1992 Jul;(280):186-97. PubMed PMID: 1611741.
CONFIDENTIAL
119
• A 25-year-old Veteran is planning to have an elbow contracture
release. His elbow range of motion is 40 degrees to 90 degrees of
flexion; his girlfriend recently left, so he currently has no help with
perineal hygiene. He has no heterotopic ossification. His ring and
small fingers become numb as his elbow approaches his flexion
endpoint. There is no evidence of instability of the ulna-humeral or
radioulnar joints. To achieve the best possible outcome, the
surgeon should
• 1- include postsurgical elbow continuous passive motion (CPM).
2- perform the surgery open.
3- decompress the ulnar nerve.
4- release the anterior band of the medial collateral ligament.
CONFIDENTIAL
120
•
PREFERRED RESPONSE: 3- decompress the ulnar nerve.
•
•
DISCUSSION
The patient is exhibiting signs of ulnar neuropathy. The surgeon should be sure to decompress and
possibly transpose the ulnar nerve, if unstable, to prevent worsening neuropathy after surgery.
CPM has not been shown to be of benefit after contracture release. Equal success rates have been
shown for open and arthroscopic contracture releases. The anterior band of the medial collateral
ligament is important to maintain valgus stability of the elbow. The posterior band can be released
to improve flexion without increasing concern for elbow instability.
•
•
RECOMMENDED READINGS
Charalambous CP, Morrey BF. Posttraumatic elbow stiffness. J Bone Joint Surg Am. 2012 Aug
1;94(15):1428-37. doi: 10.2106/JBJS.K.00711. Review. PubMed PMID: 22854997.
Williams BG, Sotereanos DG, Baratz ME, Jarrett CD, Venouziou AI, Miller MC. The contracted elbow:
is ulnar nerve release necessary? J Shoulder Elbow Surg. 2012 Jun 26. [Epub ahead of print]
PubMed PMID: 22743068.
Lindenhovius AL, van de Luijtgaarden K, Ring D, Jupiter J. Open elbow contracture release:
postoperative management with and without continuous passive motion. J Hand Surg Am. 2009
May-Jun;34(5):858-65. Epub 2009 Apr 11. PubMed PMID: 19362791.
CONFIDENTIAL
121
• What is the most appropriate definitive treatment in a 65year-old man who has experienced symptoms for more
than 1 year and has a partial-thickness rotator cuff tear
involving 90% of the tendon and arthroscopy shown?
• 1- Rotator cuff and scapular stabilizer strengthening
exercises
2- Diagnostic and therapeutic corticosteroid injection
3- Arthroscopic debridement
4- Completion of rotator cuff tear, repair, and biceps
tenotomy
5- Repair of rotator cuff and superior labrum anterior to
posterior (SLAP) repair
CONFIDENTIAL
122
• PREFERRED RESPONSE: 4- Completion of
rotator cuff tear, repair, and biceps tenotomy
CONFIDENTIAL
123
• What is the most appropriate treatment for a 25-yearold man 1 week after falling off a ladder? His axial T2weighted MRI scan is shown.
• 1- Rotator cuff and scapular stabilizer strengthening
exercises
2- Diagnostic and therapeutic corticosteroid injection
3- Arthroscopic debridement
4- Completion of rotator cuff tear, repair, and biceps
tenotomy
5- Repair of rotator cuff and superior labrum anterior
to posterior (SLAP) repair
6- Repair of subscapularis
tendon and biceps tenodesis 124
CONFIDENTIAL
•
PREFERRED RESPONSE: 6- Repair of subscapularis tendon and biceps tenodesis
•
•
DISCUSSION
Initial treatment of partial-thickness rotator cuff tears should be nonsurgical, with a focus on
rehabilitative exercises. Stiffness is more common after rotator cuff repair with concomitant SLAP
repair, and SLAP repair is not advocated in most people older than 40 years of age. Rotator cuff
repair with biceps tenotomy or tenodesis is preferred to a SLAP repair in this patient. Figure 82
shows a complete tear of the subscapularis tendon with medial subluxation of the biceps tendon. In
young patients, acute repair is preferred with stabilization of the biceps tendon.
•
•
RECOMMENDED READINGS
Oh JH, Kim SH, Kwak SH, Oh CH, Gong HS. Results of concomitant rotator cuff and SLAP repair are
not affected by unhealed SLAP lesion. J Shoulder Elbow Surg. 2011 Jan;20(1):138-45. doi:
10.1016/j.jse.2010.04.003. Epub 2010 Jul 15. PubMed PMID: 20634099.
Wolff AB, Sethi P, Sutton KM, Covey AS, Magit DP, Medvecky M. Partial-thickness rotator cuff tears.
J Am Acad Orthop Surg. 2006 Dec;14(13):715-25. PubMed PMID: 17148619.
CONFIDENTIAL
125
Figure 83a
Figure 83b
Figures 83a and 83b are the radiographs of a 53-year-old otherwise healthy homemaker who had a syncopal episode and sustained a
ground-level fall and injury to her right elbow. She presently admits to right elbow pain, swelling, and an inability to bend her elbow.
What is the best initial treatment for this injury?
1- Closed reduction with immobilization
2- Closed reduction with percutaneous pinning
3- Open reduction, bicolumnar fixation with plate and screws
4- Open reduction, bicolumnar fixation with Kirschner wires
CONFIDENTIAL
126
•
PREFERRED RESPONSE: 3- Open reduction, bicolumnar fixation with plate and screws
•
•
DISCUSSION
The radiographs and CT scans indicate a comminuted and displaced intra-articular fracture of the distal humerus.
Rigid internal fixation with bicolumnar orthogonal or parallel plating is the treatment of choice for most fractures
of the distal humerus that involve the joint surface. Closed reduction and variations thereof will not yield a stable
environment for healing. To achieve adequate exposure for fixation, a chevron olecranon osteotomy is the
preferred approach. Disadvantages associated with this approach include complications such as nonunion of the
osteotomy site and intra-articular adhesions. Prominent hardware may need to be removed during a secondary
procedure, and intraoperative conversion to an elbow arthroplasty may be limited. The most common
complications after open reduction and internal fixation include elbow stiffness, nonunion (2%-10%), and ulnar
neuropathy (0%-12%).
•
•
RECOMMENDED READINGS
Galano GJ, Ahmad CS, Levine WN. Current treatment strategies for bicolumnar distal humerus fractures. J Am
Acad Orthop Surg. 2010 Jan;18(1):20-30. Review. PubMed PMID: 20044489.
Ring D, Gulotta L, Chin K, Jupiter JB. Olecranon osteotomy for exposure of fractures and nonunions of the distal
humerus. J Orthop Trauma. 2004 Aug;18(7):446-9. PubMed PMID: 15289692.
Sanchez-Sotelo, J. Open reduction and internal fixation of supracondylar and intercondylar fractures. In: Wiesel
SW, ed. Operative Techniques in Orthopaedic Surgery. Philadelphia, PA: Lippincott Williams & Wilkins; 2011:33293336.
•
CONFIDENTIAL
127
Figure 84
CLINICAL SITUATION FOR QUESTIONS 84 THROUGH 87
Figure 84 is the glenoid CT scan of a 20-year-old man who dislocated his shoulder anteriorly while playing football. He had
persistent instability 2 months after the injury, but he did not have a sulcus sign or posterior instability. He underwent an
arthroscopic Bankart repair with 4 bioabsorbable anchors with simple sutures through the labrum and capsule. He did not have
an engaging Hill-Sachs lesion, the rotator cuff was unremarkable, and the capsule was not torn from the humerus. After surgery,
he did well for 6 months until he jumped into a lake and again dislocated his shoulder anteriorly. He says his shoulder no longer
felt stable after his reduction.
Question 84
What is the most likely reason this patient’s arthroscopic Bankart repair failed?
1- The surgeon did not use enough anchors to repair the labrum.
2- The surgeon did not recognize significant bone loss of the anterior glenoid.
3- The patient returned to full activity too soon.
4- The patient has unrecognized multidirectional instability.
CONFIDENTIAL
128
• PREFERRED RESPONSE: 2- The surgeon did not
recognize significant bone loss of the anterior
glenoid.
CONFIDENTIAL
129
• Question 85
• This patient would like to return to football and
perform normal activities of daily living without
worrying about another dislocation. What
treatment would you recommend?
• 1- Open Bankart repair
2- Coracoid transfer
3- Revision arthroscopic labrum repair
4- Arthroscopic pan capsular plication and labrum
repair
CONFIDENTIAL
130
• PREFERRED RESPONSE: 2- Coracoid transfer
CONFIDENTIAL
131
• Question 86
• What is the most common early complication
of the revision procedure for this patient?
• 1- Loss of external rotation
2- Loss of internal rotation
3- Recurrent instability
4- Subscapularis tear
CONFIDENTIAL
132
• PREFERRED RESPONSE: 1- Loss of external
rotation
CONFIDENTIAL
133
• Question 87
• What is the most common late complication
of the revision procedure for this patient?
• 1- Glenohumeral arthritis
2- Bone graft absorption
3- Anterior ligament attenuation
4- Rotator cuff tear
CONFIDENTIAL
134
•
PREFERRED RESPONSE: 1- Glenohumeral arthritis
•
•
DISCUSSION
The CT scan shows bone loss exceeding 20% on the anterior glenoid, which is the most likely reason the
arthroscopic Bankart repair failed. One study showed that using 3 or fewer anchors increases risk for failure; 4
anchors were used in this patient, so that is not the likely cause of failure. The patient returned to full activity 6
months after surgery, which is the usual time needed to regain full strength in the shoulder and ensure complete
labrum healing. Suture configuration has not been shown to affect failure rates. The patient did not have signs of
multidirectional instability such as a sulcus sign on examination, instability without a labral tear, or excessive
translation of the humeral head posteriorly on examination. This patient has recurrent instability due to glenoid
bone loss, so the procedure of choice would need to restore the anterior bone to the glenoid. The coracoid
transfer procedure uses the coracoid for bone restoration, but iliac crest bone graft would be appropriate as well.
An open Bankart repair, arthroscopic capsular plication, or a revision arthroscopic repair are all soft-tissue
procedures, which do not correct the bone loss. Braces may work to allow a patient to finish a season before
having surgery, but will not allow a return to activities of daily living without instability. The most common
complication of the coracoid transfer is a loss of external rotation. The rate of recurrent instability is low. Most
patients regain all of their internal rotation. The technique for the coracoid transfer splits the subscapularis
muscle, so a tear of the muscle is rare. Axillary or musculocutaneous nerve palsies are rare after this procedure,
but can occur if the nerves are not protected and mobilized during the dissection of the conjoint tendon. The most
common long-term complication is early arthritis of the glenohumeral joint. Most cases of arthritis are
asymptomatic and appear on follow-up radiographs. The graft rarely absorbs, and tears of the rotator cuff are
uncommon with this procedure and infection is rare. Anterior ligament attenuation is uncommon, and some
surgeons do not even repair the anterior labrum or capsule because this can lead to a loss of external rotation
after surgery.
CONFIDENTIAL
135
•
•
•
•
•
•
•
RECOMMENDED READINGS
Burkhart SS, De Beer JF. Traumatic glenohumeral bone defects and their relationship to failure of
arthroscopic Bankart repairs: significance of the inverted-pear glenoid and the humeral engaging
Hill-Sachs lesion. Arthroscopy. 2000 Oct;16(7):677-94. PubMed PMID: 11027751.
Ochoa E Jr, Burkhart SS. Glenohumeral bone defects in the treatment of anterior shoulder
instability. Instr Course Lect. 2009;58:323-36. PubMed PMID: 19385546.
Schmid SL, Farshad M, Catanzaro S, Gerber C. The Latarjet procedure for the treatment of
recurrence of anterior instability of the shoulder after operative repair: a retrospective case series
of forty-nine consecutive patients. J Bone Joint Surg Am. 2012 Jun 6;94(11):e75. doi:
10.2106/JBJS.K.00380. PubMed PMID: 22637215.
Neyton L, Young A, Dawidziak B, Visona E, Hager JP, Fournier Y, Walch G. Surgical treatment of
anterior instability in rugby union players: clinical and radiographic results of the Latarjet-Patte
procedure with minimum 5-year follow-up. J Shoulder Elbow Surg. 2012 Dec;21(12):1721-7. doi:
10.1016/j.jse.2012.01.023. Epub 2012 May 5. PubMed PMID: 22565042.
Hovelius L, Vikerfors O, Olofsson A, Svensson O, Rahme H. Bristow-Latarjet and Bankart: a
comparative study of shoulder stabilization in 185 shoulders during a seventeen-year follow-up. J
Shoulder Elbow Surg. 2011 Oct;20(7):1095-101. doi: 10.1016/j.jse.2011.02.005. Epub 2011 May 24.
PubMed PMID: 21602067.
Hovelius L, Saeboe M. Neer Award 2008: Arthropathy after primary anterior shoulder dislocation-223 shoulders prospectively followed up for twenty-five years. J Shoulder Elbow Surg. 2009 MayJun;18(3):339-47. doi: 10.1016/j.jse.2008.11.004. Epub 2009 Feb 28. PubMed PMID: 19254851.
CONFIDENTIAL
136
• Complete transection of the ulnar nerve at the
elbow will result in
• 1- loss of sensation on the ulnar side of the
index finger.
2- weakness with thumb extension.
3- weakness with elbow flexion.
4- weakness with finger abduction.
CONFIDENTIAL
137
• PREFERRED RESPONSE: 4- weakness with finger abduction.
• DISCUSSION
• Ulnar nerve lesions manifest with weakness in the finger abductor
muscles. There will be loss of interossei muscle function as well as the
third and fourth lumbricals. Extensor pollicis longus function is based on
the posterior interosseous nerve (radial), not the ulnar. The index finger
has sensation from the median nerve distribution. Elbow flexion strength
is not dependent on the ulnar nerve.
• RECOMMENDED READINGS
• Hoppenfeld S. Physical Examination of the Spine and Extremities. New
York: Appleton-Century-Crofts; 1976:93-104.
• Netter FH. The CIBA Collection of Medical Illustrations. Musculoskeletal
System Part 1. Vol 8. Summit, NJ: Ciba-Geigy Corporation; 1987:28.
CONFIDENTIAL
138
Figure 89a
Figure 89b
Figures 89a and 89b are the radiograph and MRI scan of a 40-year-old man who fell down a flight of stairs. His upper
arm is bruised and painful, and global weakness in the shoulder girdle function is noted. A radiograph is ordered to rule
out a fracture or dislocation. You should recommend
1- immediate open reduction and internal fixation of the fracture.
2- closed treatment with serial radiographs.
3- fracture fragment excision and deltoid repair.
4- rest, ice, anti-inflammatory medications, and a home exercise program.
CONFIDENTIAL
139
•
PREFERRED RESPONSE: 4- rest, ice, anti-inflammatory medications, and a home exercise program.
•
•
DISCUSSION
The patient has an os acromiale. The type shown is of the meso-acromion. This is not an acute
fracture; well corticated ends are seen on the axillary radiograph and there is no bone edema on
the T2 axial MRI image. A trial of nonsurgical care that includes rest, ice, and anti-inflammatory
medication is recommended. If a patient continues to have symptoms, an arthroscopic evaluation is
needed to determine if the os is mobile and if os fixation is appropriate.
•
•
RECOMMENDED READINGS
Sammarco VJ. Os acromiale: frequency, anatomy, and clinical implications. J Bone Joint Surg Am.
2000 Mar;82(3):394-400.
Harris JD, Griesser MJ, Jones GL. Systematic review of the surgical treatment for symptomatic os
acromiale. Int J Shoulder Surg. 2011 Jan;5(1):9-16.
Abboud JA, Silverberg D, Pepe M, Beredjiklian PK, Iannotti JP, Williams GR, Ramsey ML. Surgical
treatment of os acromiale with and without associated rotator cuff tears. J Shoulder Elbow Surg.
2006 May-Jun;15(3):265-70.
•
CONFIDENTIAL
140
Figure 90
Question 90 of 105
Figure 90 is the initial radiograph of a 28-year-old woman who sustained an acute right elbow injury. Following closed treatment under
sedation in the emergency department, the elbow is seen to be stable through an arc from full flexion down to 30 degrees short of full
extension, while the forearm is pronated but only to 75 degrees short of full extension while in supination. What structure is most
likely to remain intact?
1- Lateral ulnar collateral ligament
2- Radial head
3- Posterior band of the medial collateral ligament (MCL)
4- Anterior band of the MCL
CONFIDENTIAL
141
•
PREFERRED RESPONSE: 4- Anterior band of the MCL
•
•
DISCUSSION
The most common pattern of elbow dislocation is associated with posterolateral rotatory instability.
This pattern begins with valgus, axial load, and supination rotating the radial head posterior with
respect to the capitellum and failure of the lateral ulnar collateral ligament. The posterior band of
the MCL tears next, and the anterior band of the MCL is last to fail. In elbows with an intact anterior
band of the MCL, forearm pronation will place this structure under tension and assist in
maintaining joint reduction. If this band is torn, pronation will lead to medial joint space widening.
Radial head fractures, along with coronoid fractures, are common associated injuries, as seen in the
radiographs for this patient.
•
•
RECOMMENDED READINGS
O'Driscoll SW, Morrey BF, Korinek S, An KN. Elbow subluxation and dislocation. A spectrum of
instability. Clin Orthop Relat Res. 1992 Jul;(280):186-97.
Olsen BS, Søjbjerg JO, Nielsen KK, Vaesel MT, Dalstra M, Sneppen O. Posterolateral elbow joint
instability: the basic kinematics. J Shoulder Elbow Surg. 1998 Jan-Feb;7(1):19-29
O'Driscoll SW. Classification and evaluation of recurrent instability of the elbow. Clin Orthop Relat
Res. 2000 Jan;(370):34-43. Review.
CONFIDENTIAL
142
• A 68-year-old right-hand-dominant man underwent a right total shoulder
arthroplasty (TSA) 3 months ago. He was started on passive range of
motion and started active motion 6 weeks after surgery. He notes that he
fell onto his outstretched right arm 2 weeks ago but did not seek care. His
primary symptom is poor active elevation of the right shoulder. His right
shoulder motion has active elevation of 45 degrees, passive elevation of
140 degrees, 95-degree external rotation, and internal rotation to L3. His
left shoulder has active and passive elevation of 160 degrees, external
rotation of 70 degrees, and internal rotation to T12. The right shoulder
radiographs show a concentric total shoulder arthroplasty with no
fractures or other abnormalities. What is the most appropriate treatment
at this point?
• 1- Reassurance and a review of his rehabilitation program with an
emphasis on deltoid strengthening
2- Open repair of the subscapularis tendon
3- Latissimus dorsi tendon transfer
4- Revision to reverse TSA
CONFIDENTIAL
143
•
PREFERRED RESPONSE: 2- Open repair of the subscapularis tendon
•
•
DISCUSSION
This patient had a fall approximately 2½ months following a TSA. He now has poor
active elevation but good passive motion and external rotation exceeding that of
the contralateral shoulder. Rupture of the subscapularis tendon, which would have
been released and repaired intrasurgically, would be the primary concern in this
scenario. In the native shoulder treated surgically for instability, subscapularis
failure can produce pain, weakness of abdominal press and lumbar pushoff,
apprehension, and frank instability. Further delay in treatment of the tendon
failure with therapy is not indicated because this will lead to further muscle
atrophy and adhesions to the scapula and overlying brachial plexus. Augmentation
or replacement with a transfer of the superior portion of the pectoralis major
muscle is sometimes required. Transfer of the pectoralis minor muscle is also
described. However, latissimus dorsi transfer is described for irreparable
supraspinatus deficiency. Revision to a reverse TSA can be considered as a salvage
of a persistently unstable shoulder, but will not be the primary treatment for this
shoulder if radiograph findings are normal.
CONFIDENTIAL
144
• RECOMMENDED READINGS
• Lazarus MD, Harryman DT II. Open repairs for anterior instability. In:
Warner J, Iannotti J, Gerver R, eds. Complex and Revision Problems
in Shoulder Surgery. Philadelphia, PA: Lippincott-Raven; 1996:47-63.
• Sanchez-Sotelo J, Sperling JW, Rowland CM, Cofield RH. Instability
after shoulder arthroplasty: results of surgical treatment. J Bone
Joint Surg Am. 2003 Apr;85-A(4):622-31.
• Moeckel BH, Altchek DW, Warren RF, Wickiewicz TL, Dines DM.
Instability of the shoulder after arthroplasty. J Bone Joint Surg Am.
1993 Apr;75(4):492-7.
Miller BS, Joseph TA, Noonan TJ, Horan MP, Hawkins RJ. Rupture of
the subscapularis tendon after shoulder arthroplasty: diagnosis,
treatment, and outcome. J Shoulder Elbow Surg. 2005 SepOct;14(5):492-6.
CONFIDENTIAL
145
Figure 94
CLINICAL SITUATION FOR QUESTIONS 94 THROUGH 96
Figure 94 is the anteroposterior radiograph of a 75-year-old woman who has a 5-year history of progressive pain, crepitus, and loss of
motion in her shoulder. She had a rotator cuff repair 10 years ago. Examination reveals 60 degrees of active forward elevation and 20
degrees of external rotation with her arm at her side. Passively she can be brought to 160 degrees of forward elevation and 90
degrees of external rotation with her arm at her side. A glenohumeral joint injection with local anesthetic eliminated pain, but there is
no observed change in active motion.
Question 94
Based upon the information provided, you should recommend
1- total shoulder arthroplasty (TSA).
2- arthroscopic rotator cuff repair.
3- arthroscopic debridement.
4- reverse total shoulder arthroplasty (rTSA).
CONFIDENTIAL
146
• PREFERRED RESPONSE: 4- reverse total
shoulder arthroplasty (rTSA).
CONFIDENTIAL
147
• Question 96
• A common postoperative radiographic
observation associated with your surgery in an
asymptomatic patient is
• 1- implant fracture.
2- suture anchor dislodgement.
3- scapular notching.
4- acromial fracture.
CONFIDENTIAL
148
•
PREFERRED RESPONSE: 3- scapular notching.
•
•
DISCUSSION
The patient has anterior superior instability. This condition is caused by a
combination of rotator cuff insufficiency and loss of coracoacromial arch integrity.
The only known way to restore shoulder function in this scenario is to implant a
rTSA. The device works by converting the translational force of the deltoid into a
rotational force, resulting in restoration of forward elevation of the shoulder.
Performing a rotator cuff repair or arthroscopic debridement will not address this
biomechanical problem. TSA will also not change this biomechanical problem. The
poor motion and function are not a result of synovitis or pain because an injection
with local anesthetic has eliminated the pain and serves as a useful test to
determine if rTSA is the only viable solution. If the patient can achieve nearnormal function with a local anesthetic challenge, rTSA is overtreatment. Scapular
notching is a long-term concern for implant longevity because it represents bone
loss under the baseplate of the glenoid component. This loss of support can lead
to catastrophic failure of the device. Implant fracture, acromial fracture, and
dislodgement of suture anchors are not likely to be asymptomatic in a non-Charcot
joint.
CONFIDENTIAL
149
•
•
RECOMMENDED READINGS
Melis B, DeFranco M, Ladermann A, Mole D, Favard L, Nerot C, Maynou C, Walch G. An evaluation
of the radiological changes around the Grammont reverse geometry shoulder arthroplasty after
eight to 12 years. J Bone Joint Surg Br. 2011 Sep;93(9): 1240-6. PubMed PMID: 21911536
Leung B, Horodyski M, Struk AM, Wright TW. Functional outcome of hemiarthroplasty compared
with reverse total shoulder arthroplasty in the treatment of rotator cuff tear arthropathy. J Shoulder
Elbow Surg. 2012 Mar;21(3):319-23. Epub 2011 Aug 26.
Cheung E, Willis M, Walker M, Clark R, Frankle MA. Complications in reverse total shoulder
arthroplasty. J Am Acad Orthop Surg. 2011 Jul;19(7):439-49. Review.
Walker M, Brooks J, Willis M, Frankle M. How reverse shoulder arthroplasty works. Clin Orthop
Relat Res. 2011 Sep;469(9):2440-51. Review.
Sadoghi P, Vavken P, Leithner A, Hochreiter J, Weber G, Pietschmann MF, Müller PE. Impact of
previous rotator cuff repair on the outcome of reverse shoulder arthroplasty. J Shoulder Elbow
Surg. 2011 Oct;20(7):1138-46. Epub 2011 Mar 30.
Nam D, Kepler CK, Neviaser AS, Jones KJ, Wright TM, Craig EV, Warren RF. Reverse total shoulder
arthroplasty: current concepts, results, and component wear analysis. J Bone Joint Surg Am. 2010
Dec;92 Suppl 2:23-35.
CONFIDENTIAL
150
• A 36-year-old woman dislocated her elbow 6
months ago. The elbow was congruently reduced
and rehabilitated. She continues to have a sense
of painful clunking in her elbow when she pushes
up from a chair with forearm supination, but not
pronation. What structure did not heal properly?
• 1- Posterior band of the medial collateral
ligament
2- Anterior band of the medial collateral ligament
3- Radial collateral ligament
4- Lateral ulnar collateral ligament
CONFIDENTIAL
151
•
PREFERRED RESPONSE: 4- Lateral ulnar collateral ligament
•
•
DISCUSSION
The patient is showing signs of posterolateral rotatory instability after elbow
dislocation. The lateral ulnar collateral ligament is responsible for stabilizing the
elbow against this type of instability. The posterior and anterior bands of the
medial collateral ligament are primarily resistors of valgus load in elbow extension
and flexion, respectively. The radial collateral ligament does not control the
posterolateral rotatory instability described.
•
•
RECOMMENDED READINGS
O'Driscoll SW, Morrey BF, Korinek S, An KN. Elbow subluxation and dislocation. A
spectrum of instability. Clin Orthop Relat Res. 1992 Jul;(280):186-97
View Abstract at PubMed
O'Driscoll SW, Bell DF, Morrey BF. Posterolateral rotatory instability of the elbow. J
Bone Joint Surg Am. 1991 Mar;73(3):440-6.
•
CONFIDENTIAL
152
• What complication following total elbow
arthroplasty poses more risk for a 60-year-old
man with osteoarthritis than for a man of the
same age with rheumatoid arthritis?
• 1- Aseptic loosening of a linked implant
2- Instability of an unlinked implant
3- Triceps rupture
4- Wound dehiscence
CONFIDENTIAL
153
•
PREFERRED RESPONSE: 1- Aseptic loosening of a linked implant
•
•
DISCUSSION
Patients with elbow osteoarthritis tend to be active and are often involved in manual occupations that place
greater demands on a total elbow implant. Such patients are most often treated with nonprosthetic options
because of concerns about prosthetic longevity. As a result, few cases of primary osteoarthritis are included in
published studies. However, complications such as stem fracture and aseptic loosening appear to be more
common in this population than in any other subgroup, including revision patients. The poor soft-tissue quality
associated with rheumatoid arthritis leads to a high-risk ligamentous attenuation and is a general contraindication
to use of an unlinked implant. The same poor soft tissue leads to a higher rate of triceps insufficiency and wound
dehiscence.
•
•
RECOMMENDED READINGS
Kozak TKW, Adams RA, Morrey BF. Total elbow arthroplasty for primary osteoarthritis. In: Morrey BF, SanchezSotelo J, eds. The Elbow and Its Disorders. Philadelphia, PA: WB Saunders; 2009:843-848.
•
Gill DRJ, Morrey BF, Adams RA. Linked total elbow arthroplasty in patients with rheumatoid arthritis. Total elbow
arthroplasty for primary osteoarthritis. In: Morrey BF, Sanchez-Sotelo J, eds. The Elbow and Its Disorders.
Philadelphia, PA: WB Saunders; 2009:782-791.
•
Gramstad GD, Galatz LM. Management of elbow osteoarthritis. J Bone Joint Surg Am. 2006 Feb;88(2):421-30.
Review.
Gill DR, Morrey BF. The Coonrad-Morrey total elbow arthroplasty in patients who have rheumatoid arthritis. A ten
to fifteen-year follow-up study. J Bone Joint Surg Am. 1998 Sep;80(9):1327-35.
CONFIDENTIAL
154
Figure 99a
Figure 99b
Figure 99a is the radiograph of a 48-year-old woman 8 months after initial treatment of an injury. She initially was placed in a
sling and progressive rehabilitation followed. She now has refractory pain but normal range of movement and strength. The
current radiograph is shown in Figure 99b. The most appropriate next treatment step is
1- Resumption of sling immobilization
2- Open reduction and internal fixation
3- Application of a bone stimulator and rest from exacerbating activities
4- Application of a figure-of-8 brace
CONFIDENTIAL
155
•
PREFERRED RESPONSE: 2- Open reduction and internal fixation
•
•
DISCUSSION
The radiographs reveal an atrophic nonunion of the midshaft of the clavicle. The treatment of acute, displaced
midshaft clavicle fractures in adults continues to evolve, with several reports advocating early surgical
intervention. Although many fractures heal, symptomatic delayed unions or nonunions occur and may eventually
require treatment. In this case, further sling immobilization or use of a figure-of-8 brace is unlikely to lead to
fracture consolidation at 8 months after the injury. Although use of an electrical bone stimulator may be
attractive, there is no conclusive data suggesting its efficacy in promoting healing of a displaced clavicular
nonunion. Most authors advocate treatment with open reduction internal plate fixation. Controversy exists as to
the need for allograft or autograft bone augmentation.
•
•
•
•
RECOMMENDED READINGS
van der Meijden OA, Gaskill TR, Millett PJ. Treatment of clavicle fractures: current concepts review. J Shoulder
Elbow Surg. 2012 Mar;21(3):423-9. doi: 10.1016/j.jse.2011.08.053. Epub 2011 Nov 6. Review.
Khan LA, Bradnock TJ, Scott C, Robinson CM. Fractures of the clavicle. J Bone Joint Surg Am. 2009 Feb;91(2):44760. doi: 10.2106/JBJS.H.00034. Review.
Endrizzi DP, White RR, Babikian GM, Old AB. Nonunion of the clavicle treated with plate fixation: a review of fortyseven consecutive cases. J Shoulder Elbow Surg. 2008 Nov-Dec;17(6):951-3. doi: 10.1016/j.jse.2008.05.046. Epub
2008 Sep 20.
CONFIDENTIAL
156
• A 75-year-old Veteran sustained a 4-part fracture dislocation of the
proximal humerus with a comminuted humeral head. You decide to
perform a reverse total shoulder replacement because of age and activity
level, and because it is so cool. This will be your first reverse total shoulder
replacement. It is common practice in your hospital for an industry
representative to be present when new implants are brought into the
operating room. What information are you required to disclose?
• 1- This is an experimental procedure.
• 2-You are “the second greatest surgeon the world has ever seen.”
2- You have no financial relationship with the implant company, but wish
you did.
3- There will be an implant company representative in the room.
4- The hospital will lose money because of the cost of the implant.
• 5-Your attending will be in the OR heckling you while you are performing
the procedure.
CONFIDENTIAL
157
•
PREFERRED RESPONSE: 3- There will be an implant company representative in the room.
•
•
DISCUSSION
Current recommendations from the American Orthopaedic Association Orthopaedic Institute of
Medicine are that the patient be notified if an industry representative is going to be present in the
operating room. This surgery is not experimental for this indication, and Medicare currently covers
the surgery for patients with appropriate indications. Court cases have demonstrated that surgeonrelated factors can be litigated (such as surgeon experience), but there are no current requirements
to disclose this. Surgeons are not required to disclose cost and compensation information to their
patients.
•
•
RECOMMENDED READINGS
Report from the task force on surgeon-industry relationships in the discipline of orthopaedic
surgery. American Orthopaedic Association Orthopaedic Institute of Medicine Task Force on
Surgeon-Industry Relationships in the Discipline of Orthopaedic Surgery. J Bone Joint Surg Am. 2012
Jun 20;94(12):e89.
Bal BS, Choma TJ. Clin Orthop Relat Res. What to disclose? Revisiting informed consent. 2012
May;470(5):1346-56.
CONFIDENTIAL
158
G
o
o
d
l
u
c
CONFIDENTIAL
159